Clinical Reasoning Exam 2

¡Supera tus tareas y exámenes ahora con Quizwiz!

What prediction scoring systems are use to determine if a pt who has pneumonia can be treated safely as an outpt or if hospitalization is required Which scoring system is the most validated?

--CURB65 mortality prediction tool -*the pneumonia severity index (PSI/PROT score)* (MOST VALIDATED)

When is the chicken pox vaccination given

-12-15 months -booster at 4 years

What is a good initial test if you suspect a pt has a lung neoplasm?

-CXR

How do you screen for TB

-PPD, CXR, TB gold (blood test)

When should you not give live vaccines in AIDS pts

-When the CD4 cound falls to less than 200 or if there is a hx of oropharyngeal candidiasis

What is the time requirement to be dx wiht chronic bronchitis? Acute bronchitis?

-a cough and sputum production on most days for at least 3 month in at least 2 consecutive years -5 days or less

If when treating someone for their GERD symptoms, H2 receptor antagonists and PPIs do not work, what is the next step in treatment what are some SE of this medication

-a gastric motility stimulant such as *metoclopromide* -irreversible dyskinesia

What is the primary presenting symptoms of COPD

-dyspnea

What pts is shingles more commonly found in

-elderly -immunosuppressed

What are the symptoms of typhoid fever

-maculopapular rash on lower chest and abdomen *"rose spots"* -fever, myalgias, diarrhea, abdominal pain, and hepatosplenomegaly

What are some rare but serious complications of AOM

-mastoiditis -temporal bone -osteomyelitis -facial nerve paralysis

What medication is given for pts with mild COPD? Moderate COPD? Severe COPD? Very Severe COPD?

-mild = SABA -moderate = LABA + SABA as needed -severe = LABA + ICS + SABA as needed -Very severe = consider surgical intervention

How long does an chronic cough last? What are some causes

-more than 8 weeks -COPD -carcinoma -upper airway cough syndrome -GERD -asthma

What organism causes trichomoniasis

-trichomonas vaginalis

What is the most common mechanism triggering pneumonia

-upper airway colinization by potentially pathogenic organisms

How does gonorrhea typically present in men? In women?

-urethral infection including dysuria and penile discharge -asymptomatic until complications like PID occur

How does trichomoniasis present

-usually asymptomatic -itching, burning or frothy green discharge -*strawberry cervix*

What are the sites of extrapulmonary spread of TB from most frequent to least frequent

1. lymph nodes 2. pleura 3. genitourinary tract 4. bones and joints 5. meninges 6. peritoneum

When performing a methacholine challenge, a change in FEV1 by ______%is considered a positive diagnosis of asthma

20

What % of people are unaware that they are infected with AIDS

25

How long does community acquired penumonia therapy last

5-10 days or until the pt is afebrile for at least 72 hours

More than _____% of nodules measuring less than 8 mm are benign

99

The fat soluble vitamin deficieny from CF may manifest as neuropathy and hemolytic anemia, night blindness, or mucosal bleeding due to the lack of what 4 vitamins

A, E, D, K

What may improve the treatment response in TB meningitis when combined with abx

Adjunctive glucocorticoids

On a CBC, what signifies a bacterial infection

Bands and a significant left shift with neutrophils = bacterial infection

1. A 59-year-old man with a known history of COPD presents with worsening dyspnea. On examination, he is afebrile. His breath sounds are decreased bilaterally. He is noted to have jugular venous distension (JVD) and 2+ pitting edema of the lower extremities. Which of the following is the most likely cause of his increasing dyspnea? A COPD exacerbation B Pneumonia C Cor pulmonale D Pneumothorax

C Cor pulmonale JVD and lower extremity edema are suggestive of cor pulmonale, which is right heart failure due to chronically elevated pressures in the pulmonary circulation. Right heart failure causes increased right atrial pressures and right ventricular end-diastolic pressures, which then lead to liver congestion, jugular venous distension, and lower extremity edema.

A 38-year-old woman presents with progressively worsening dyspnea and cough. She has never smoked cigarettes, has no known passive smoke exposure, and does not have any occupational exposure to chemicals. Pulmonary function testing shows obstructive lung disease that does not respond to bronchodilators. Which of the following is the most likely etiology? A Radon exposure at home B COPD C α1-Antitrypsin deficiency D Asthma

C α1-Antitrypsin deficiency This patient has a fixed airway obstruction consistent with COPD. The airway obstruction of asthma would be at least partially reversible on testing with a bronchodilator. α1-Antitrypsin deficiency should be considered in a patient who develops COPD at a young age, especially if there is no other identifiable risk factor.

A 68-year-old patient of your practice with known COPD has pulmonary function testing showing an FEV1 of 40% predicted has been having frequent exacerbations of his COPD. His SaO2 by pulse oximetry is 91%. Which of the following medication regimens is the most appropriate? A. Inhaled salmeterol BID and albuterol as needed B. Oral albuterol daily and inhaled fluticasone BID C. Inhaled fluticasone BID, inhaled tiotropium BID, and inhaled albuterol as needed D. Inhaled fluticasone BID, inhaled tiotropium BID, inhaled albuterol as needed, and home oxygen therapy

C. Inhaled fluticasone BID, inhaled tiotropium BID, and inhaled albuterol as needed This patient has stage III COPD with frequent exacerbations. He is best treated by a long-acting bronchodilator (eg, tiotropium) and an inhaled steroid (eg, fluticasone) used regularly, along with an inhaled, short-acting bronchodilator on an as-needed basis.

What medication would be used for aspiration pneumonia?

Clindamycin

What is bronchiectasis

Condition in which a bronchus or bronchi remains dilated after an infection or obstruction.

which STI is also common to be nonsexually transmitted in children

HSV

What is the treatment for latent TB

INH for 9 months with a goal of preventing reactivation TB later in life

What condition does hemoptysis wiht an acute onset of pleuritis CP and dyspnea suggest

PE

What is the most effective therapy for prompt relief of asthmatic syptoms

SABA like *albuterol, levalbuterol, and pirbuterol*

T or F: Cough caused by ACE inhibitors can be triggered after the first dose or may occur after months of therapy.

T

a. Each of the following factors increases her risk for developing otitis media EXCEPT: A) She attends day care. B) Her mother smokes inside the house. C) The patient is a female. D) Patient still uses a pacifier.

The correct answer is "C." The following are known risk factors for the development of AOM: day care attendance, smoking inside the home, male gender, pacifier use, children in developing countries, age between 6 and 18 months, and lack of breast feeding.

Which of the following is/are considered first-line treatment for otitis externa? A) Oral ciprofloxacin. B) Acetic acid ear drops. C) Polymyxin and neomycin combination ear drops. D) A and C. E) B and C.

The correct answer is "E." Otitis externa can be treated with a wide array of topical agents. One option is to acidify the external ear canal. Neither Pseudomonas nor Staphylococcus species can thrive at an acidic pH. Thus, acetic acid drops (VoSol®) can be used: they are cheap and effective. Another approach is to use a topical antibiotic. Polymyxin/neomycin combinations (e.g., Cortisporin) are safe and effective. A number of other antibiotic preparations are available as well. Alcohol-based solutions are another alternative. "A" is incorrect because oral treatment is not indicated for simple otitis externa. However, topical ciprofloxacin may be used.

1. A 10-year-old Caucasian boy has a history of recurrent sinusitis and multiple episodes of pneumonia. You suspect CF and order a sweat chloride test. The sweat electrolyte test result is within the normal range. What is your next step in management? a. Perform DNA testing for CFTR (cystic fibrosis transmembrane conductance regulator) gene mutations. b. Perform a pH probe test for gastroesophageal reflux. c. Referral to pulmonologist. d. Reassure parents that he does not have CF. e. Place him on a high-calorie, high-protein diet.

The correct answer is A. You answered A. Explanation: A. The child has recurrent upper and lower respiratory tract infections suggesting CF. The sweat chloride test can yield falsely low values, so the next step would be to perform DNA testing to identify any of the common CFTR mutations. Negative sweat chloride test results do not exclude CF.

What score is used to estimate the likelihood of a PE

Wells score

What type of lung cancer has the least association with smoking and a stronger association with pulmonary scar/fibrosis

adenocarcinoma

Reactivation of TB usually involves what lobes of the lungs

apical and posterior segments of the upper lobes

What is the purpose of pneumatic otoscopy

applying slight positive adn negative pressure with a rubber bulb to verify TM mobility

How is pertussis treated

azithromycin

Most pts with hemoptysis require evaluation with what method

bronchoscopy

1. A 35-year-old woman complains of calf tenderness and acute dyspnea. The arterial blood gas reveals partial pressure of oxygen (PO2) of 76 mm Hg. Which of the following is the most common physical examination finding of pulmonary embolism? a. Wheezing b. Increased pulmonary component of the second heart sound c. Tachypnea d. Calf swelling e. Pulmonary rales

c. Tachypnea •Tachypnea is the most common physical sign associated with pulmonary embolus. Calf or thigh pain and/or swelling occurs less frequently than tachypnea. Other common clinical manifestations of pulmonary embolus in decreasing frequency include pleuritic pain, cough, and orthopnea.

What is the best diagnostic test to diagnosis PE

chest CT with IV contrast

Pts with gonorrhea typically are also infected with what STI and should therefore be screened

chlamydia

What is the most common etiology of COPD

cigarette smoking (associated with 90% of cases)

What causes hand food and mouth disease and herpangina

coxsackie virus

1. A 54-year-old woman is noted to have cervical cancer and presents with significant vaginal bleeding with a hemoglobin level of 7 g/dL. Her left leg is swollen, which on Doppler investigation reveals a deep venous thrombosis. Which of the following is the best treatment for the thrombus? a. Intravenous unfractionated heparin b. Fractionated subcutaneous heparin c. Subcutaneous unfractionated heparin d. Oral warfarin (Coumadin) e. Vena cava filter

e. Vena cava filter Cervical cancer with significant vaginal bleeding is a relative contraindication for anticoagulation. Thus, a vena cava filter is the most appropriate choice in this patient.Bottom of Form

What causes scarlet fever

group A streptococcus

How does a pt with sepsis/shock present

hypotention

In a person who has a pelural effusion, a low ph could indiate what

infection

What criteria is used to determine if a pleural effusion can be labeled an exudate?

light criteria

What type of cancer is the leading cause of death in both men and women

lung

what is the tx for roseola

no treatment necessary

What causes erythema infectiosum/fifth disease

parbovirus B19

What is the treatment of choice for scarlet fever

penicillin -cephalosporins and macrolides are an alternative for allergic pts

What is a parapneumonic effusion

pleural effusion + pneumonia at the same time. the fluid is usually tranudative and lcear

What organism must you consider in CF pts as it is rarely found in conditions outside of CF

pseudomonas

Are RPR and VDRL more sensitive or specific for syphilis If FTA-ABS more sensitive or specific

sensitive bc its a screening test specific bc its a confirmatory test

What is the most aggressive type of lung cancer

small cell

What type of lung cancer is the most aggressive, but is also most likely to respond to chemo/radiation

small cell

What type of lung cancer is usually metastatic at the time of diagnosis and not resectable

small cell

What type of lung cancer ist he most responsive to chemo/radiation

small cell

What is the best method to monitor lung function

spirometry

What is the most common bacterial cause of the common cold

strep pneumo

What is the most common cause of community acquired pneumonia

streptococcus pneumonia

LRAs are the treatment of choice for which type of pt

those who are sensitive to asprin

Why are multiple drugs used to treat active TB

to avoid resistance

What organism cause syphilis

treponema pallidum

What is the treatment for Pneumocystis jirovecii

trimethoprim sulfamethoxazole

What anticoagulation shoudl be used immediatley in pts with PE

unfractionated heparin, LMWH, enoxaparin, or tinzaparin for the direct factor Xa inhibitor fondaparinux

What organism causes chicken pox

varicella zoster virus

What anticoagulation must you monitor the INR closely

warfarin

What lobes does primary TB usually effect

- middle and lower lobes -lesions form in the periphery with hilar and paratracheal lymphadenopathy

How can you avoid methicillin resistant s aureus (MRSA) when treating pts for pneumonia

-Avoid intubation when possible -Use oropharyngeal intubation as opposed to nasopharyngeal intubation -Keep the head of the bed elevated during tube feedings -Hand washing, alcohol based hand disinfectants

What are treatments for penumonia (FARLEY)? Which one is the most common?

-Azithromycin + Ampicillin -*Ceftriazone* (MOST COMMON) -Clindamycin for aspiration penumonia -Levofloxacin for pts with pneumonia and DM

What are some causes of transudative pleural effusions

-Congestive heart failure -Nephrotic syndrome -Cirrhosis with ascites -Myxedema (severe hypothyroidism) -Pulmonary embolism -Nephrotic syndrome

As asthma gets worse, what values decrease on spirometry

-FEV1/FVC and FEV1

In COPD, what are the values of FVC, FEV1, and the ratio of FEV1 to FVC like

-FVC and FEV1 are reduced -the ratio is less than .7, which indicates airway obstruction

What is the "treatment" of HIV/AIDS

-HAART (several antiviral drugs) -Influenza/penumococcal vaccine for prophylaxis

A 61 yr old pt present the the ER with cough for 2 weeks that is productive of green sputum and is associated with sweating, shaking chills, adn fever up to 102F. Her grandchildren had URIs 2 weeks ago but are find now. PMI includes DM x 10yrs which is controlled via hypoglycemics. Denies tobacco, alcohol or drugs. She looks ill and in distress with coughing and chills. BP is 100/80, pulse is 110, temp is 101F, respirations are 24 O2 sats is 97%. Lungs have rhonchi and decreased breath sounds with dullness to percussion in BL bases. Heart is tachycardic but regular. No cyanosis or edema. Rest of exam is normal. CBC shows high WBC count of 17,000 with 85% neutrophils and 20% lymphocytes. Blood sugar is 120 mg/dl How would you treat this pt?

-IV levofloxacin (a fluroquinolone) since she has DM as well

What are some causes of exudative pleural effusions

-Infection -Tuberculosis -Malignancy -Connective tissue disease -Pancreatitis -Chylothorax -Asbestos exposure

What medication has a synergistic effect when combined wiht a beta 2 agonist

-Ipratropium

How should empyema be treated?

-It may not respond to antibiotics if it is encapsulate so it may need drained

If a pt has otitis externa and you believe it is due to staph and they do not have RF for MRSA, what should you use as treatment

-Keflex aka Cephflexin (1st gen) or methicillin

What class of medications should never be used as a monotherapy for long term control of persistent asthma

-LABAs

What 3 organisms cause atypical pneumonia

-M. pneumoniae -chlamydia pneumoniae -Legionella pneumophila

what can cause elevation in D dimer besides a PE

-MI -pneumonia -heart failure -cancer -sepsis

What are the 2 most widely used leukotriene receptor antagonists? (LRAs) In what pts are they most effectively used

-Montelukast and fzafirlukast -Those who cannot use ICS

What is superior vena cava syndrome What are the symptoms

-Obstruction of venous drainage, usually by extrinsic compression of the SVC -edema of the face, neck, and upper part of the torso

If you find evidence of blood during a thorocentesis, what 3 things could this suggest

-PE, malignancy or trauma

What is done for initial screening for syphilis? What is done for confirmation?

-RPR or VDRL test -FTA-ABS (fluorescent treponemal antibodie absorbed)

What is the most common cause of pneumonia in children

-RSV

What 2 asthma meds, when combined, have synergistic effects in treating effects and are useful in treating severe asthma attacks

-SABA + Ipatropium

If a pleural effusion is clear yellow in color, is it transudative or exudative? What are 3 possible common causes

-Transudative -secondary to CHF, cirrhosis, nephrotic syndrome

How can you confirm the dx of chicken pox

-Tzanck smear or DNA PCR

What is secondary prevention of PE

-anticoagulation

What is the therapy for PE

-anticoagulation (heparin, warfarin) either oral or IV -tPA -IVC filter if they have a DVT

If pseudomonas is considered a likely cause of a pts pneumonia, such as in immunocompromised pts and those with recent hospitalization with intubation, what treatment should be given

-antipneumococcal and antipseudomonal beta lactam (*piperacillin/tazobactam, cefepime, imipenem, or meropenem) PLUS a fluoroquinolone (*levofloxacin, moxifloxacin) and/or an aminoglycoside (*amikacin or tobramycin and azithromycin)

What is the treatment for HSV

-antiviral therapy for acute management of symptomatic outbreaks and women who are pregnant

Pts with pneumonia caused by legionalla wil present with what distinct symptoms/signs

-electrolyte imbalances bc of diarrhea -elevated liver enzymes

What medication is use to treat pneumonia in pts with chronic comorbidities like DM or CHF

-fluroquinolones (*levofloxacin, moxifloxacin) -combo of beta lactam (*high dose amoxicillin, amoxicillin/clavulanate, cefpodoxime, or cefuroxime) PLUS a macrolide (*clarithromycin or azithromycin*)

What is the presentation of chlamydia

-frequently asymptomatic so screening is vital

A 68-year-old woman is brought to the emergency center after coughing up several tablespoons of bright red blood. For the previous 3 to 4 months, she has had a chronic nonproductive cough but no fevers. More recently, she has noticed some scant blood-streaked sputum. On review of her symptoms, she reports increased fatigue, decreased appetite, and a 25-lb weight loss in the past 3 months. She denies chest pain, fever, chills, or night sweats. The patient has smoked one pack of cigarettes per day for the past 35 years. She drinks two martinis every day and has not had any significant medical illness. She worked in a library for 35 years and has no history of occupational exposures. She does not take any medication except for one aspirin per day. The patient is a thin woman who is mildly anxious, alert, and oriented. Her blood pressure is 150/90 mm Hg, heart rate is 88 bpm, respiratory rate is 16 bpm, and temperature is 99.2°F. Neck examination reveals no lymphadenopathy, thyromegaly, or carotid bruit. The chest has scattered rhonchi bilaterally, but there are no wheezes or crackles. Cardiovascular examination reveals a regular rate and rhythm, without rubs, gallops, or murmurs. The abdomen is benign with no hepatosplenomegaly. Examination of her extremities reveals no cyanosis; there is finger clubbing. Neurologic examination is normal. What is the most likely dx

-lung ca

What are the symptoms of erythema infectiosum/fifth disease

-mild fever and upper respiratory symptoms before outbreak of rash for 4-14 days -confluent erythematous macules on the mace that spares the nose and periorbital regions *slapped cheek* -facial rash is followed by a lacy reticular rash on the trunk and extremities

How will pts with pneumocystis jiroveci present

-nonproductive cough, fever, dyspnea that worsens -febrile, tachypneic, and hypoxic -*ground glass on CXR*

After a pt is infected with HIV, how may they initially present

-nonspecific symptoms such as low grade fever, fatigue adn sore throat or myalgias 6-8 weeks after initial infection

What does the CXR of a COPD pt look like

-normal until the disease is advanced -hyperinflation of the lungs -bullae (parenchymal destruction)

When would you get an ABG for a COPD pt

-not responding quickly -altered mental state -low PaCO2

When should you use a methacholine challenge test in a pt? Why

-only use if the dx is in doubt -it provokes a histamine like response and could cause an asthma attact

What is the treatment of trichomoniasis

-oral metronidazole -tinidazole is an alternative

Obstruction to the pulmonary artery causes platelets to release vasoconstrictive agents such as ______, which cause a ________ (depression/elevation) in vascular resistance

-serotonin -elevation

Lung cancers can be divided into what 2 major categories

-small cell lung cancer -non small cell lung cancer

A 59-year-old male presents with a 3-week history of hoarseness. He denies sore throat or heartburn. He has had no fevers, night sweats, or weight loss. When he initially presented a week ago, your partner treated him empirically for postnasal drainage. He smokes 2 packs of cigarettes per day and drinks alcohol daily. On exam, his vital signs are normal. His voice sounds husky. You find no other abnormalities. What risk factors increase the likelihood that it is laryngeal cancer

-smoking, alcohol use, long length of symptoms

What organism causes toxic shock syndrome

-staphylococcus aureus

A 60-year-old man is diagnosed with moderately severe (stage II) COPD. He admits to a long history of cigarette smoking and is still currently smoking. In counseling him about the benefits of smoking cessation, which of the following statements is most accurate? A. By quitting, his pulmonary function will significantly improve. B. By quitting, his current pulmonary function will be unchanged, but the rate of pulmonary function decline will slow. C. By quitting, his current pulmonary function and the rate of decline are unchanged, but there are cardiovascular benefits. D. By quitting, his pulmonary function will approach that of a nonsmoker of the same age.

B. By quitting, his current pulmonary function will be unchanged, but the rate of pulmonary function decline will slow. Smoking cessation will not result in reversal of the lung damage that has already occurred, but can result in a slowing in the rate of decline of pulmonary function. In fact, smoking cessation can result in the rate of decline returning to that of a nonsmoker.

If a young child has recurrent cases of sinusitis, what should you consider in your ddx

CF

T or F: the varicella vaccine is a live, attenuated virus and should not be given to certain pts

T

T or F: there is now a recommended meningococcal vaccine

T

T or F: you can add a LABA without ICS in COPD but not in asthma pts

T

t or F: a normal ore near normal CXR is the most common finding in PE

T

-T or F: elderly pts often have fewer or less severe symptoms or atypical presentation of penumonia

T -pneumonia is in the ddx if and elderly pt presents with altered mental status

A 2-year-old is brought to your office by her mother who is concerned that she has been pulling at her left ear since late last night and has a fever of 101.3°F. She has had recurrent bouts of these symptoms, the last of which was 9 months ago. Each time, the symptoms resolved with one "shot." She is alert and interactive. She has some evidence of mucoid discharge from her nares bilaterally. What is the most appropriate treatment for this patient? A) Amoxicillin 40 mg/kg/day divided TID. B) Amoxicillin 80-90 mg/kg/day divided BID. C) Ceftriaxone 50 mg/kg IM once. D) Azithromycin 10 mg/kg for 1 day then 5 mg/kg for days 2-5. E) Amoxicillin/clavulanate 40-80 mg/kg/day divided BID.

The correct answer is "B." Amoxicillin is the first-line treatment of AOM. The dose is 80-90 mg/kg/day in all patients whether antibiotic naïve or not. More broad-spectrum (and expensive) drugs such as ceftriaxone and amoxicillin/clavulanate should be reserved for patients who fail initial therapy with a first-line drug or have a penicillin allergy.

A 59-year-old male presents with a 3-week history of hoarseness. He denies sore throat or heartburn. He has had no fevers, night sweats, or weight loss. When he initially presented a week ago, your partner treated him empirically for postnasal drainage. He smokes 2 packs of cigarettes per day and drinks alcohol daily. On exam, his vital signs are normal. His voice sounds husky. You find no other abnormalities. The best next step in the management of this patient is: A) Empiric antibiotic treatment. B) Empiric proton pump inhibitor treatment. C) Direct laryngoscopy. D) Esophagogastroduodenoscopy (EGD). E) Neck MRI.

The correct answer is "C." The first concern is to rule out malignancy, so the larynx should be visualized. Direct laryngoscopy is a straightforward office procedure and takes only a few minutes. If the equipment and expertise are not available in your office, referral to an otolaryngologist is appropriate. Although there are no firm guidelines, some authors recommend direct laryngoscopy after 2 weeks of hoarseness in patients who are at risk (older patients and those who have a history of tobacco and alcohol use). Since laryngoscopy is such an available, low-cost, low-risk procedure, it is hard to justify postponing it for any patient at risk for malignancy. Therefore, "A" and "B" are incorrect, as further empiric medication trials will only delay laryngoscopy. Besides, antibiotics are not indicated for the lone symptom of hoarseness. In other instances, empiric proton pump inhibitor therapy may be more practical, since gastroesophageal reflux is a common cause of hoarseness. "D" is incorrect. In this case, direct laryngoscopy is preferred to EGD. Finally, "E" is incorrect. Neck MRI is not indicated in the initial evaluation of hoarseness, but it might be used for follow-up after laryngoscopy or to investigate a neck mass.

A 42-year-old businesswoman presents to your office with the chief complaint of 2 days of headache, sore throat, and nasal congestion productive of green mucus. She denies any fever, contact with ill persons, and gastrointestinal symptoms, but she does have a history of seasonal allergies. On exam, she has completely normal vital signs. Her posterior oropharynx has mild erythema and postnasal drainage but no exudates. There is nasal mucosal erythema and swelling with clear rhinorrhea. Her neck is supple with no adenopathy. Respirations are clear. Initial treatment for this patient includes: A) Oral decongestants. B) NSAIDs. C) Oral antibiotics. D) A and B. E) A and C.

The correct answer is "D." Most cases of rhinosinusitis are viral and need only symptomatic treatment. Analgesics and systemic and nasal decongestants are reasonable options. Other treatment options include ipratropium nasal spray (which will also help to decrease mucus production), nasal steroids (at least one well-done randomized trial proved mometasone is superior to placebo), and first-generation antihistamines with anticholinergic activity (e.g., diphenhydramine).

This patient is concerned about recurrences of her otitis externa. What advice can you give her? A) Avoid exposure by putting a petroleum jelly (e.g., Vaseline) impregnated cotton plug in her ear before swimming. B) Use a blow dryer on her ear after swimming. C) Instill a 50/50 mixture of alcohol and vinegar in her ears after swimming. D) Avoid swimming when she has active disease. E) All of the above.

The correct answer is "E." All of the above can be used to minimize disease recurrence. The benefit of "C" is less certain.

a. Which of the following findings is reliably found in patients with AOM? A) Fever. B) Ear pulling. C) Irritability. D) Rhinitis. E) None of the above.

The correct answer is "E." None of the above is reliably found in patients with AOM. Other unreliable factors include vomiting, diarrhea, and cough. The presence or absence of any of these findings is nothelpful in making the diagnosis of otitis media. Note that while ear pain is a symptom of inflammation, it is a relatively weak predictor of AOM and must be accompanied by other findings as listed above. However, pneumatic otoscopy or otoscopy plus tympanometry is the way to make the diagnosis, but always make sure that you have a good seal or you run the risk of a "false-positive" finding. This patient has not had any problems with otitis media for at least 9 months, has not been on antibiotics during that time, is not in day care, and has no allergies. You opt to treat her with an antibiotic.

1. A 1-month-old boy has a fever to 102.7°F (39.3°C), is irritable, has diarrhea, and has not been eating well. On examination he has an immobile red TM that has pus behind it. Which of the following is the most appropriate course of action? a. Admission to the hospital with complete sepsis evaluation b. Intramuscular ceftriaxone and close outpatient follow-up c. Oral amoxicillin-clavulanate d. Oral cefuroxime e. High-dose oral amoxicillin

The correct answer is A. A. Very young children with OM (especially if irritable or lethargic) are at higher risk for bacteremia or other serious infection. Hospitalization and parenteral antibiotics often are needed.

Which of the following tests are the most important to follow for a patient receiving isoniazid and rifampin for tuberculosis treatment? A Renal function tests B Liver function tests C Slit-lamp examinations D Amylase and lipase tests

The correct answer is B. Explanation: Drug-induced liver injury is a complication of treatment with isoniazid, pyrazinamide, and rifampin. Baseline liver tests are obtained in all patients and monthly monitoring of hepatic enzymes is recommended for patients at increased risk of liver toxicity. Alcohol use, prior liver disease, pregnancy, and the first 3 months postpartum are risk factors.

A 4-year-old boy is brought to your office by his mother for evaluation of a rash on his face that his mother first noticed the day prior to presentation. His mother comments that it looks like somebody "slapped him." The mother reports that he has had a cold for the last couple of days. The child's physical examination is unremarkable except for an erythematous macular rash over both cheeks. The mother admits that the child is behind on his immunization schedule. Which of the following is the most likely cause? A. Varicella-zoster virus B. Parvovirus B19 C. Human herpes virus 6 D. Rubella virus E. Child abuse and you should contact social services immediately

The correct answer is B. You answered B. Explanation: This question describes erythema infectiosum, or fifth disease, which is caused by parvovirus B19. It often has a prodrome of fever and upper respiratory systems mistaken by the mother in this question as a "cold." This child also has the classic "slapped cheek" rash of erythema infectiosum and, while the child does need to be caught up on his immunizations, the child has the classic symptoms of fifth disease and not of the diseases for which children are immunized.

1. A 3-month-old infant is admitted to the hospital with her third episode of lobar pneumonia and wheezing. Findings that would increase your suspicions for CF and prompt sweat chloride testing include all of the following EXCEPT: a. Hyponatremia, hypochloremia, and metabolic alkalosis b. Failure to thrive c. Lymphocytosis d. Digital clubbing e. Oily appearing stools

The correct answer is C. You answered C. Explanation: C. Infants with CF will lose excess amounts of sodium chloride in their sweat resulting in a hyponatremic, hypochloremic metabolic alkalosis. Malabsorption of fats and protein due to pancreatic exocrine insufficiency usually presents as steatorrhea and is a major cause of morbidity for patients with CF, resulting in failure to thrive. Lymphocytosis can occur with viral infections or pertussis but is not a hallmark of CF.

You are on duty when an 18-year-old man is brought to the emergency room (ER) from his college dorm by his roommate. He is confused and cannot give a history. He has a temperature of 104.0°F (40.0°C), pulse of 110 beats/min, blood pressure of 90/60 mm Hg, and a respiration rate of 24 breaths/min. His head cannot be moved because of severe nuchal rigidity. Multiple petechiae are observed on his buttocks and legs. What is the most appropriate advice to give to this patient's roommate? A. Reassurance that he does not require prophylaxis. B. Take acyclovir for prophylaxis. C. Take penicillin for prophylaxis. D. Take rifampin for prophylaxis. E. Take cefuroxime for prophylaxis.

The correct answer is D. You answered D. Explanation: The patient has meningitis and meningococcemia caused by N meningitidis. The patient is severely affected and is in septic shock. All people in close contact with the patient should receive ciprofloxacin or rifampin prophylaxis.

What should be given to a child with AOM if they demonstrate clinical failure after 3 treatment days

amoxicillin-clavulanate, cefuroxime axetil, cefdinir, azithromycin, ceftriaxone, or tympanocentesis is considered

1. A 39-year-old man is noted to have a deep venous thrombosis without any known risk factors. He notes that his brother also developed a pulmonary embolism at age 45, and his mother developed a "clot in the leg" when she was in her thirties. Which of the following is the most likely inherited disorder in this patient? a. Protein S deficiency b. Antithrombin III deficiency c. Factor V Leiden mutation d. Antiphospholipid antibody syndrome e. Familial malignancy syndrome

c. Factor V Leiden mutation Factor V Leiden mutation is the most common hereditary thrombophilia. It is inherited in an autosomal dominant fashion and therefore will affect both men and women.

What is the most frequently reported STI in the US

chlamydia

a. What is your next diagnostic step? A) Pneumatic otoscopy. B) Hearing test. C) Tympanostomy. D) No further diagnosis needed—treat with antibiotic. T

he correct answer is "A." Even though we all think we can do it well, the diagnosis of otitis media is fraught with problems. Pneumatic otoscopy should be done in essentially all patients but especially in those in whom long-term therapy is being considered. Remember that fluid can persist for a month or more after an otitis media. On your exam, the TM does not move with insufflation. The patient's mother asks you if the child should have tubes placed.

SIADH leads to ______

hyponatremia

In CF, the loss of the endocrine glands of the skin causes a ____________, __________ alkalosis.

hyponatremic, hypochloremic

What is a exudative effusion? What are the LDH and protein levels like? What are some common causes

•Effusion caused by inflammatory or malignant causes, usually with high protein or high LDH levels. oCauses include cancer, infection, PE

A 39-year-old homeless man presents to the emergency department (ED) with a nonproductive cough and subjective fever. He says that his illness has been worsening over the past 2 weeks, originally starting with dyspnea on exertion and now he is short of breath at rest. On questioning, he tells you that he lives in a homeless shelter when he can, but he frequently sleeps on the streets. He has used IV drugs (primarily heroin) "on and off" and has been sexually promiscuous with both men and women without barrier protection for many years. He denies any significant medical history, and only gets medical care when he comes to the ED for an illness or injury. On review of systems, he complains of chronic fatigue, weight loss, and diarrhea. On examination, he is a thin, disheveled man appearing much older than his stated age. His temperature is 100.4°F (38.0°C), his blood pressure is 100/50 mm Hg, his pulse is 105 beats/min, and his respiratory rate is 24 breaths/min. His initial oxygen saturation is 89% on room air, which comes up to 94% on 4 L of oxygen by nasal cannula. Significant findings on examination include dry mucous membranes, a tachycardic but regular cardiac rhythm, a soft and nontender abdomen, and generally wasted-appearing extremities. His pulmonary examination is significant for tachypnea and fine crackles bilaterally, but there are no visible signs of cyanosis on extremities. His chest x-ray reveals diffuse, bilateral, interstitial infiltrates that look like "ground glass." What diagnostic testing and treatment should be started?

-*CBC with differential* to look for low lymphocytes, leukopenia -*CMP* to check kidney/liver infection (Hep B is commonly sexually transmitted, Hep C could be from IV drug use) -*CD4 count* -*UA to check for STIs (chlamydia, gonorrhea) or a urethra swab not clean catch -echo (TEE) to look for infective endocardidis due to IV drug use esp of the tricuspid valve. Check using an It would most likely be caused by staph aureus

A 37-year-old man presents to your office with the complaint of cough. The cough began approximately 3 months prior to this appointment, and it has become more annoying to the patient. The cough is nonproductive and worse at night and after exercise. He has had a sedentary lifestyle but recently started an exercise program, including jogging, and says he is having a much harder time with the exertion. He runs out of breath earlier than he did to previously, and he coughs a lot. He has not had any fever, blood-tinged sputum, or weight loss. He denies nasal congestion and headaches. He does not smoke and has no significant medical history. His examination is notable for a blood pressure of 134/78 mm Hg and lung findings of occasional expiratory wheezes on forced expiration. A chest radiograph is read as normal. What is the most likely diagnosis What else could be on the ddx

-*asthma* -bronchitis/post bronchitis -pertussis -bronchiectasis -sarcoidosis -ACE inhibitors -CHF -occupational irritants -COPD -GERD -allergies

What is the treatment for gonorrhea

-*ceftriaxone* -treatment for chalmydia (*azithromycin or doxycycline*)

If a pleural effusionbloody and the HCT of the pleural fluid is >1/2 of the Hct in the peripheral blood, what are some likely causes? What if its <1/2 the Hct in the blood

-*hemothroax* due to trauma, rupture of blood vessel or malignancy -cancer, PE, TB

What tests are preferred to test for latent TB in those who have the BCG vaccine as they are not affected by the vaccine

--Quantiferon TB Gold assay and the T SPOT TB assay

When performing a peak flow before and after bronchodilator, a change in FEV1 by ______% or _______ ml within ________ minutes classifies it as reversible and therefore asthma

-12, 200, 15

What is the treatment for TB

-2 month course of 4 drug treatment with isoniazid (INH), rifampin, pyrazinamide, and ethambutol followed by 4 months of INH and rifampin

How long does hospital acquired penumonia cause by s aureus, psuedomonas aeruginosa, kelbsiella, anaerobes,, m pneumoniae, C pneumoniaae or legionella

-2 to 3 weeks

If a pt remains symptomatic After maximal medical treatment for GERD, what is the next step in their treatment to confirm the dx

-24 hour esophageal pH monitioring -esophagogastroduodenoscopy showing esophagitis

How many samples should be taken to dx TB? At least one sample should be taken when?

-3 samples -morning

How long does an subacute cough last? What are some causes

-3-8 weeks -Viruses such as RSV, influenza and adenovirus -bacteria such as pertussis, chlamydia, or mycoplasma

When initiating warfarin therapy, the usual course is to use UFH, LMWH, for fondaparinux for at least _____ days while overlapping with warfarin until ___________________

-5 -the INR has been therapeutic for 2 consecutive days

What IV steroids can be given to pts having a COPD exacerbation? How quickly do they act

-60 mgs methylprednesone, or dexamethasone 10 mg, -Steroids typically act within several hours

What are potential complications of penumonia

-Acute respiratory distress syndrome (ARDS) -sepsis/shock -ventilator or bipap may be needed -pleural effusion -empyema -DKA if she has DM1 -Hyperosmolar Hyperglycemic Nonketoic syndrome if she has DM2

On CXR, atypical organisms that cause pneumonia thend to cause ______________ infiltrates rathen than _________ infiltrates

-BL, diffuse -focal, lobar

What are some HIV related conditions that are not AIDS Defining

-Bacillary angiomatosis • Oropharyngeal candidiasis •Persistent, recurrent, or difficult-to-treat vaginal candidiasis •Cervical dysplasia or carcinoma in situ •Oral hairy leukoplakia •Idiopathic thrombocytopenic purpura •Listeriosis •Pelvic inflammatory disease (especially if complicated by tubo-ovarian abscess) •Peripheral neuropathy •Herpes zoster, two or more episodes involving more than one dermatome

A patient with known asthma undergoing therapy with inhaled corticosteroids and intermittent (short-acting) β2-agonist presents with complaints of nocturnal awakenings secondary to cough and occasional wheezing. This episode occurs three to four times per week. Pulmonary function tests in the past have shown mild obstructive lung disease. Which of the following is the best next step? a. Oral steroids b. Leukotriene inhibitors c. Long-acting β2-agonists d. Theophylline e. Antireflux therapy

-C. long acting B2 agonists Long-acting β2-agonists are indicated in this situation. The asthma would be classified as moderate persistent, and the recommended treatment is addition of long-acting β2-agonists (such as salmeterol) to the inhaled corticosteroids, which are particularly helpful with nocturnal symptoms.

A 68-year-old woman is brought to the emergency center after coughing up several tablespoons of bright red blood. For the previous 3 to 4 months, she has had a chronic nonproductive cough but no fevers. More recently, she has noticed some scant blood-streaked sputum. On review of her symptoms, she reports increased fatigue, decreased appetite, and a 25-lb weight loss in the past 3 months. She denies chest pain, fever, chills, or night sweats. The patient has smoked one pack of cigarettes per day for the past 35 years. She drinks two martinis every day and has not had any significant medical illness. She worked in a library for 35 years and has no history of occupational exposures. She does not take any medication except for one aspirin per day. The patient is a thin woman who is mildly anxious, alert, and oriented. Her blood pressure is 150/90 mm Hg, heart rate is 88 bpm, respiratory rate is 16 bpm, and temperature is 99.2°F. Neck examination reveals no lymphadenopathy, thyromegaly, or carotid bruit. The chest has scattered rhonchi bilaterally, but there are no wheezes or crackles. Cardiovascular examination reveals a regular rate and rhythm, without rubs, gallops, or murmurs. The abdomen is benign with no hepatosplenomegaly. Examination of her extremities reveals no cyanosis; there is finger clubbing. Neurologic examination is normal. What is the next step

-CMP too look for liver/kidney dysfunction -CBC to look at hematocrit for anemia, WBC for infection, and platelets to see if they can clot -PT/PTT to see if they can clot -TSH to r/o hyperthyroidism -CXR/CT

A 52-year-old man presents because of coughing and SOB. He has had multiple office visits in the past few years for similar reasons. He has a chronic "smoker's cough," but reports that in the past 2 days his cough has increased, his sputum has changed from white to green in color, and he has had to increase the frequency with which he uses his albuterol inhaler. He denies having a fever, CP, peripheral edema, or other symptoms. His medical history is significant for hypertension, peripheral vascular disease, and two hospitalizations for pneumonia in the past 5 years. He has a 60-pack-year history of smoking and continues to smoke two packs of cigarettes a day. On examination, he is in moderate respiratory distress. His temperature is 98.4°F, his blood pressure is 152/95 mm Hg, his pulse is 98 beats/min, his respiratory rate is 24 breaths/min, and he has an oxygen saturation of 94% on room air. His lung examination is significant for diffuse expiratory wheezing and a prolonged expiratory phase of respiration. There are no signs of cyanosis. The remainder of his examination is normal. A chest x-ray done in your office shows an increased anteroposterior (AP) diameter and a flattened diaphragm, but otherwise he has clear lung fields What is the most likely dx?.

-COPD

A 52-year-old man presents because of coughing and SOB. He has had multiple office visits in the past few years for similar reasons. He has a chronic "smoker's cough," but reports that in the past 2 days his cough has increased, his sputum has changed from white to green in color, and he has had to increase the frequency with which he uses his albuterol inhaler. He denies having a fever, CP, peripheral edema, or other symptoms. His medical history is significant for hypertension, peripheral vascular disease, and two hospitalizations for pneumonia in the past 5 years. He has a 60-pack-year history of smoking and continues to smoke two packs of cigarettes a day. On examination, he is in moderate respiratory distress. His temperature is 98.4°F, his blood pressure is 152/95 mm Hg, his pulse is 98 beats/min, his respiratory rate is 24 breaths/min, and he has an oxygen saturation of 94% on room air. His lung examination is significant for diffuse expiratory wheezing and a prolonged expiratory phase of respiration. There are no signs of cyanosis. The remainder of his examination is normal. A chest x-ray done in your office shows an increased anteroposterior (AP) diameter and a flattened diaphragm, but otherwise he has clear lung fields What is the DDX?

-COPD -neoplasm -bronchitis -pneumonia -PE -CHF -MI -liver failure -GERD -Anemia -Pneumothorax -foreign body -allergies -asthma -metabolic acidosis -ACE inhibitors -valvular disorder -anxiety

A 48-year-old woman calls 911 and is brought to the emergency center complaining of a sudden onset of dyspnea. She reports she was standing in the kitchen making dinner, when she suddenly felt as if she could not get enough air, her heart started racing, and she became light-headed and felt as if she would faint. She denied chest pain or cough. Her medical history is significant only for gallstones, for which she underwent a cholecystectomy 2 weeks previously. The procedure was complicated by a wound infection, requiring her to stay in the hospital for 8 days. She takes no medications regularly, and only takes acetaminophen as needed for pain at her abdominal incision site. On examination, she is tachypneic with a respiratory rate of 28 bpm, oxygen saturations 84% on room air, heart rate 124 bpm, and blood pressure 118/89 mm Hg. She appears uncomfortable, diaphoretic, and frightened. Her oral mucosa is slightly cyanotic, her jugular venous pressure is elevated, and her chest is clear to auscultation. Her heart rhythm is tachycardic but regular with a loud second sound in the pulmonic area, but no gallop or murmur. Her abdominal examination is benign, with a clean incision site without signs of infection. Her right leg is moderately swollen from mid-thigh to her feet, and her thigh and calf are mildly tender to palpation. Laboratory studies including cardiac enzymes are normal; her electrocardiogram (ECG) reveals only sinus tachycardia, and her chest x-ray is interpreted as normal.Top of Form What is the most appropriate diagnostic step

-CT angiogram of the chest (contrast) -D Dimer -Venous US of LE to look for DVT -V/Q scan (ventilation perfussion mismatch)

A 52-year-old man presents because of coughing and SOB. He has had multiple office visits in the past few years for similar reasons. He has a chronic "smoker's cough," but reports that in the past 2 days his cough has increased, his sputum has changed from white to green in color, and he has had to increase the frequency with which he uses his albuterol inhaler. He denies having a fever, CP, peripheral edema, or other symptoms. His medical history is significant for hypertension, peripheral vascular disease, and two hospitalizations for pneumonia in the past 5 years. He has a 60-pack-year history of smoking and continues to smoke two packs of cigarettes a day. On examination, he is in moderate respiratory distress. His temperature is 98.4°F, his blood pressure is 152/95 mm Hg, his pulse is 98 beats/min, his respiratory rate is 24 breaths/min, and he has an oxygen saturation of 94% on room air. His lung examination is significant for diffuse expiratory wheezing and a prolonged expiratory phase of respiration. There are no signs of cyanosis. The remainder of his examination is normal. A chest x-ray done in your office shows an increased anteroposterior (AP) diameter and a flattened diaphragm, but otherwise he has clear lung fields What other tests could you order

-CXR -CBC with differential to r/o anemia, infection •ABG, Especially if there is hypoxia, hypercapnia, or altered mental state •CMP to r/o liver disease and look at kidney function

A 61 yr old pt present the the ER with cough for 2 weeks that is productive of green sputum and is associated with sweating, shaking chills, adn fever up to 102F. Her grandchildren had URIs 2 weeks ago but are find now. PMI includes DM x 10yrs which is controlled via hypoglycemics. Denies tobacco, alcohol or drugs. She looks ill and in distress with coughing and chills. BP is 100/80, pulse is 110, temp is 101F, respirations are 24 O2 sats is 97%. Lungs have rhonchi and decreased breath sounds with dullness to percussion in BL bases. Heart is tachycardic but regular. No cyanosis or edema. Rest of exam is normal. CBC shows high WBC count of 17,000 with 85% neutrophils and 20% lymphocytes. Blood sugar is 120 mg/dl What is your next diagnostic step

-CXR -blood cultures -BMP (BUN and creatinine) -sputum culture -ABGs and HgbA1C if appropriate (not in this case)

In more than 90% of cases, a normal CXR in an immunocompetent nonsmoker guides the physician to make one of which 3 diagnoses?

-GERD -asthma -upper airway cough syndrome

If GERD does not get better with lifestyle modifications, what should be the next step in treatment

-H2 receptor antagonist like *famotidine or ranitidine* -PPI lik *omeprazole*

What causes Roseola

-HHV6 (human herpes virus 6)

What would you look for on exam if you suspected a pneumothorax

-Hyperpresence to percussion -decreased breath sounds -tracheal deviation

In pts with persistent asthma, what 3 classes of medications should be considered

-ICS -leukotriene receptor antagonists -LABAS

What treatment would you give pts with community acquired pneumonia who do not require ICU treatment

-IV beta lactam (*cefotaxime, ceftriaxone, or ampicillin/sulbactam) PLUS and IV macrolide (*erythromycin or azithromycin*)

A 37-year-old man presents to your office with the complaint of cough. The cough began approximately 3 months prior to this appointment, and it has become more annoying to the patient. The cough is nonproductive and worse at night and after exercise. He has had a sedentary lifestyle but recently started an exercise program, including jogging, and says he is having a much harder time with the exertion. He runs out of breath earlier than he did to previously, and he coughs a lot. He has not had any fever, blood-tinged sputum, or weight loss. He denies nasal congestion and headaches. He does not smoke and has no significant medical history. His examination is notable for a blood pressure of 134/78 mm Hg and lung findings of occasional expiratory wheezes on forced expiration. A chest radiograph is read as normal. How would you confirm the diagnosis

-Peak flow before and after bronchodilator -spirometry -methacholine challenge -CXR

What is the treatment for syphilis? What is an alternative

-Penicillin G IM or IV -doxycycline, tetracycline, or ceftriaxone used for pts who are allergic

What type of organism if more commonly seen in pts with AIDS? What will the CXR look like

-Pneumocystis jiroveci -ground glass appearing infiltrate

Large cell cancer tends to cause what condition/symptoms

-SVC syndrome -hoarsness as a consequence of laryngeal nerve paralysis

A 2½-year-old boy comes to your clinic for the first time with complaints of fever and increasing "wet" cough for 8 days. His mother reports that he has been diagnosed with asthma and has an albuterol inhaler to use for wheezing or cough. Since 6 months of age, he has had several similar episodes of "wet" cough and fever, which were diagnosed as bronchitis or pneumonia, and he would improve when treated with antibiotics and albuterol. However, over the past year, these episodes have become more frequent and the cough occurs almost daily now. Sometimes the mother sees him expectorate the sputum, which is thick and purulent. He has daily nasal congestion for which she uses saline and bulb suction in his nares. She is able to obtain some thick yellow discharge but the symptoms mainly improve when he is treated with antibiotics. He is not in daycare and has no tobacco exposure. She is concerned that his frequent illnesses are causing him to be "small for his age." The mother notes his stools are malodorous, and since starting him on potty-training she has observed that his stools float and sometimes appear to have drops of oil on them. Your examination reveals a moderately ill-appearing child whose height and weight are at the third percentile for age. His temperature is 101°F (38.3°C) and respiratory rate is 32 breaths/min. He is breathing with his mouth open. Over the upper lung fields, he has crackles and rhonchi and also a few expiratory wheezes over all lung fields. He has no heart murmur; S1 and S2 are normal. His fingers show clubbing. You obtain a chest radiograph that shows linear opacities in a parallel tram-track configuration in the upper lobes with some ring-shaped opacities; the radiologist interprets the findings as bronchiectasis. What is the next step in evaluation

-Sweat chloride test: if negative, repeat once. If negative 2 times do a CFTR genetic testing -must have respiratoyr, GI or GU symptoms as well

What are some conditions that occur more commonly in pts with AIDS

-TB -pneumococcal pnumonia -candida infections -Kaposi sarcoma -*cytomegalovirus* -*toxoplasmosis* -retinitis -cryptococcal meningitis -primary central nervous system lymphoma -HIV related dementia adn HIV associated nephropathy

What anticoagulation requires continuous infusion and laboratory monitoring

-UFH

If a pt cannot have a Chest Ct, what other test is often helpful at determining if a pt has a PE

-V/Q scan

What medication is used to treat pneumonia in healthy persons suitable for outpatient treatment

-a macrolide (*clarithromycin or azithromycin) -doxycycline

What is the cause of TB? How is it typically transmitted

-acid fastbacillus (AFB) M tuberculosis -through airborne spread of droplets from infected patients with pulmonary TB

end expiratory wheezing suggests what? localized wheezing may be consistent with what 2 things

-active bronchospasm -foreign body or bronchogenic tumor

What is the most common lung cancer

-adenocarcinoma

Atypical pneumonias occur more commonly in what pts

-adolescents or young adult pts

the pneumonia severity index (PSI/PROT score) assigns pts to a risk category based on what factors?

-age -comorbid illnesses -specific examination findings -labratory findings

What a pt is dyspneic, you should evaluate their ABC's. What does this stand for?

-airway, breathing, circulation

What organisms are involved in health care associated penumonia

-all organisms that cause community acquired penumonia -pseudomonas -Klebsiella -Acinetobacter -staphylococcus aureus

Who should be screened for syphilis

-all pregnant women to lower the risk of congenital syphilis -anoyone with another STI -those who are high risk

Pneumococcal vaccines are recommended for which pts

-all pts 65 and older -all adults with chronic cardiopulmonary diseases -Cigarette smokers -all immunocompromised pts

How does a pt with acute respiratory distress syndrome present

-altered mentation which could signal hypoxia or respiratory hypercapnia

What is the purpose of tampanometry

-an exam that measures the transfer of acoustic energy at varying levels of ear canal pressures, which will reflect TM mobility

What is the treatment for shingles When should it be started

-antiviral therapy *acyclovir or valacyclovir* for 7-10 days -within 3 days of onset of rash to reduce postherpetic neuralgia

What is the treatment for chicken pox When should treatment be started

-antiviral therapy with *acyclovir or valacyclovir* may shorten the course of the illness in pts >2 -must be started with 24 hours of exanthem

What will the CXR look like in pts with TB

-apical consolidation

What are some complications of erythema infectiosum/fifth disease

-aplastic crisis with anemia -leukopenia -fetal hydrops/pregnancy loss if transmitted to the baby

What are complications of chlamydia if untreated

-ascending infections like PID -ectopic pregnancy or infertility -cervicitis or epididymitis -urethritis and pharyngitis

A 25-year-old man presents to your office complaining of a 3-month history of rhinorrhea, itchy eyes, and exertional cough and wheezing. These symptoms have been progressively worsening over the past few months. His past medical history is significant for seasonal allergies to pollen and ragweed. His family history is significant only for hypertension in both parents. His siblings and children are in good health without allergies or respiratory illness. His social history is negative for smoking. He has worked as an animal laboratory technician for the last 6 months. On questioning, his symptoms were initially more severe toward the end of the work week but are now continuous. He has been taking over-the-counter antihistamines, which helped initially but have not relieved his allergic symptoms. On review of systems, he has noted hives that are less prominent now that he has been taking the antihistamines on a regular basis. On examination, his body mass index (BMI) is 23, blood pressure is 120/75 mm Hg, pulse is 72 beats/min, and respiratory rate is 18 breaths/min. His conjunctiva are injected, there is mild clear ocular discharge, and his nasal turbinates are boggy without visible polyps. His lung examination reveals prolonged inspiratory-to-expiratory ratio and end-expiratory wheezing at the bilateral bases. His heart examination is unremarkable and there is no peripheral edema. What is the most likely dx? What is the differential

-asthma -allergic rhinitis -uticaria -allergic conjunctivitis

What is the treatment for chlamydia Which treatment should not be given to pregnant women

-azithromycin (single dose) -doxycycline (BID for 1 week) -doxycycline bc it affects the development of fetal teeth/bones

What are some complications of penumonia

-bacteremia -parapneumonic pleural effusion -empyema (rare)

A 61 yr old pt present the the ER with cough for 2 weeks that is productive of green sputum and is associated with sweating, shaking chills, adn fever up to 102F. Her grandchildren had URIs 2 weeks ago but are find now. PMI includes DM x 10yrs which is controlled via hypoglycemics. Denies tobacco, alcohol or drugs. She looks ill and in distress with coughing and chills. BP is 100/80, pulse is 110, temp is 101F, respirations are 24 O2 sats is 97%. Lungs have rhonchi and decreased breath sounds with dullness to percussion in BL bases. Heart is tachycardic but regular. No cyanosis or edema. Rest of exam is normal. CBC shows high WBC count of 17,000 with 85% neutrophils and 20% lymphocytes. Blood sugar is 120 mg/dl What is the most likely diagnosis?

-bacterial penumonia -pleural effusion (due to dullness on percussion)

In a serious bacterial infection you will see what on a CBC

-bands -toxic granules

What should you look for on exam in a pt who has COPD

-barrel chest -distant heart sounds (bc of hyperinflation) -distant breath sounds -expiratory wheezes with prolonged expiration phase -anxious and tachypneic -use of accessory muscles -wheezes or rales -possible cyanosis

What are RF for drug resistant organisms that cause penumonia

-being hospitalized for more than 5 days -antibiotics within the last 90 days -immunosupression (AIDS, chemo, etc) -high rates of antibiotic resistance in the community

If the pleural effusion fluid is dark green, what should be considered

-billiothorax

What are the main complications of tPA

-bleeding (possible intracerebral hemorrhages)

What organism causes lyme disease

-borrelia burgdorferi

What condition does chronic copious sputum production suggest

-bronchiectasis

What are the most common causes of hemoptysis in the US

-bronchitis -lung cancer

How is HIV defined in terms of stages

-by the highest clinical stage in which the pt has ever qualified for

When performing a thoracentesis, you should stop if the pt experiences what?

-chest discomfort -SOB -they become hemodynamically unstable

What organism causes chlamydia

-chlamydia trachomatis

What is the oral antibiotic used to treat for pseudomonas in CF pts

-ciprofloxacin

Close contacts of someone with meningococcal infection should be offered what prophylaxis

-cirpofloxacin or rifampin

Based on the pneumonia severity index (PSI/PROT score), what classes of pts can be safely treated at home? which ones should be hospitalized

-class 1 and 2 (low risk pts) -Class 3, 4, 5, (high risk)

Primary therapy for a PE consists of what? What pts is this therapy used for

-clot dissolution -thrombolysis with tPA -removal of clot by surgical embolectomy -those with right heart failure or hypotension

What physical finding on the hands is often associated with chronic lung disease

-clubbing

How is chlamydia screened for

-collecting samples directly from the cervix, pharynx, or urethera

How is gonorrhea screened for

-collecting samples directly from the cervix, pharynx, or urethera or anus

what is the most common initial symptom of COPD Describe this symptom?

-cough -intermittent then becomes a daily occurence -cough is productive of white, thick mucus

Where is the most common site for clot formation

-deep proximal lower extremity veings

How do you diagnosis HIV/AIDS

-detection of HIV antibodies using the enzyme linked immunosorbent assay (ELISA) -Samples that are repeatedly positive must be confirmed by the *Western Blot testing*

What are the symptoms of chicken pox

-development of a rash in clusters that are papules or vesicles on a erythematous base and are in *different stages* -aka *dew drops on a rose petal* -malaise, fever, anorexia

What are the symptoms of TSS

-diffuse erythematous macular rash with peeling palms and soles -fever, vomiting, diarrhea, and hypotension

What is the treatment for lyme disease

-doxycycline -amoxicillin

What is the treatment for meningococcus before you know what the culture is?

-empiric antibiotics until the organism is isolated (*ampicillin + gentamicin for infants <30 days and vancomycin + ceftriaxone for adults*) -*treatment should not be delayed by performing a lumbar puncture

What does the rash of lyme disease look like? What are early symptoms? Later symptoms?

-erythema migrans *bulls eye rash* with central clearing -arthralgias/myalgias, *bell palsy*, aseptic meningitis, carditis, heart block -polyarthritis

What are the symptoms of measles

-erythematous maculopapular rash starting at the forehead and moving down to the body -*Koplik spots*

What are the symptoms of scarlet fever

-erythematous papular rash starting on neck and moving to trunk and extremities that feels like *sandpaper* -*strawberry tongue* -*pharyngitis* and fever -desqumation of the rash after 4-5 days of initial appearance

How often should HIV/AIDS pts be monitored via measuring CD4/CD8 and viral load? Why

-every 3-6 monts -monitor for disease staging, progression and risk of complications/opportunistic infecitons

What does a peak flow look for? What is considered normal? What 3 factors of the pt does it depend on

-evidence of obstruction ->80% -age, sex, and height

What should your PE consist of in a suspected asthma pt

-eyes and nose: examined for signs of allergies -neck: evaluated for accessory muscle use in respiratory distress, or lymphadenopathy -chest examined for hyperexpansion and hunched shoulders. -skin examined for signs of atopy, eczema, or urticaria. -heart examination can reveal tachycardia and pulsus paradoxus during exacerbations. -lung examination may reveal wheezing heard predominantly on end expiration or forced end expiration with prolongation of expiration compared to inspiration.

What are the most common inherited conditions for hypercoagulability

-factor V leiden -prothrombin gene mutations

What are signs and symptoms of TB

-fever -night sweats -malaise -weight loss -anorexia. -cough that is productive of purulent sputum and sometimes streaked with blood. -A Rasmussen aneurysm sometimes develops in proximity to a cavitary lesion as the inflammatory reaction causes thinning of the wall of an adjacent bronchial artery

What is a rare genetic cause of COPD? When should this be considered as a cause

-genetic deficiency in alpha1-antitrypsin -If the pt develops COPD before age 45, especially if they are a nonsmoker

If HPV is not cleared, what complications/conditions can it cause

-genital warts -cervical cancer -penile cancer -anay or oropharyngeal cancer

A tube thoracostomy or more aggressive drainage of a parapneumonic effusion usually is required under what 5 conditions

-gross pus (empyema), positive Gram stain or culture, glucose less than 60 mg/dL, pH less than 7.20, and loculations.

By the time dyspnea develops in a pt with COPD, lung function (as measured by FEV1) has been reduced by about _______ and the COPD has been present for _______

-half -years

When checking a pleural effusion pts hematocrit, if the hematocrit is the same as what it is in the blood, what does this indicate If it is less than what is in the blood, what might this indicate

-hemothorax -PE or malignancy

What causes shingles

-herpes zoster -reactivation of the varicella virus

If the LDH is high in a pleural effusion, does this indicate transudate or exudate? If it is low?

-high = exudate -low = transudate

If the glucose is hign in a pleural effusion, does this indicate transudate or exudate? If it is low?

-high = transudate -low = exudate

What are the symptoms of roseola

-high fever -erythematous maculopapular rash that appears suddenly on the trunk and spread rapidly to the extremities *sparing the face*

What are the symptoms of meningococcus

-high fever, hypotension -altered mental status -frank bacterial meningitis -erythematous maculopapular eruption that *does not blance with compression and progresses to form petechiae*

How is TB diagnosed?

-history/clinical picture -AFB stains or culture of specimen via a smear or tissue biopsy

What are risk factors for PE

-hormone replacement therapy -factor V leiden mutation -smoking obesity and sedentary lifestyle -long distance traveled -PVD (due to stasis) -pregnancy -paralysis -cancer

What are RF for health care associated penumonia?

-hospitalization within 90 days -home infusion therapy -dialysis -resident at a nursing home

What is virchows triad

-hypercoaguability -venous stasis -endothelial inflamation/injury

How should meningococcus pts be managed and what labs/tests should be done

-immediately hospitalized and quarantined -cerebrospinal fluid cultures collected

How is a positive PPD defined

-induration of at least 5 mm after 48 to 72 hours

What type of tick transmits lyme disease

-ixodes

What is the chief cause of poor asthma control

-lack of adherence to environmental controls and prescribed medications

How long does an acute cough last? What are some causes

-less than 3 weeks -*URI* -CHF -pneumonia -PE -allergic rhinitis -exacerbation of existing lung disease

What is a good screening tool for lung cancer? What is one of the biggest problems with this test

-low dose spiral CT -false positives

What laboratory findings should be taken into consideration when using the the pneumonia severity index (PSI/PROT score)

-low pH, low serum sodium -low hematocrit -low O2 sat -high glucose -high blood urea nitrogen (BUN) -pleural effusion

What are the lifestyle recommendations to help with GERD symptoms

-low-fat diet -elevation of the head of the bed -avoidance of offending foods (caffeine, alcohol, chocolate) -smoking cessation -weight reduction

If the CT and or V/Q scare are nondiagnostic, yet the clinical suspicion remains high, what other imaging modalities can be used

-lower extremity venous ultrasound -contrast enhanced MRI -echocardiography

What are other common bacterial causes of community acquired penumonia other than streptococcus pneumoniae

-mycoplasma pneumoniae -Haemophilus influenzae -Moraxella cararrhalis

What organism causes meningococcus

-neisseria meningitidis

What cormorbidities are considered high risk in a pt who has pneumonia

-neoplastic disease -liver disease -renal disease -CHF -DM

When should a pt wiht a COPD exacerbation be admitted to the hospital rather than discharged

-no improvement after treatments -vitals are poor (esp O2 sat remains low, low BP, high respiratory rate) -if the person lives alone

What types of cancers are more common: small cell or non small cell

-non small cell

What are PE findings in pts with otitis externa

-pain when pressing on the tragus or pulling on the pinna -erythematous, edematous ear canal -possible swollen lymph nodes

What is the presentation of herpes

-painful vesicles/ulcers

A 32-year-old woman presents to the emergency center complaining of productive cough, fever, and chest pain for 4 days. She was seen 2 days ago in her primary care physician's clinic with the same complaints, was diagnosed clinically with pneumonia, and was sent home with oral azithromycin. Since then, her cough has diminished in quantity. However, the fever has not abated, and she still experiences left-sided chest pain, which is worse when she coughs or takes a deep breath. In addition, she has started to feel short of breath when she walks around the house. She has no other medical history. She does not smoke and has no history of occupational exposure. She has not traveled outside of the United States and has no sick contacts. On physical examination, her temperature is 103.4°F, heart rate is 116 bpm, blood pressure is 128/69 mm Hg, respiratory rate is 24 bpm and is shallow. Her pulse oximetry is 94% saturation on room air. Physical examination is significant for decreased breath sounds in the lower half of the left lung fields posteriorly, with dullness to percussion about halfway up. There are a few inspiratory crackles in the mid-lung fields, and her right side is clear to auscultation. Her heart is tachycardic but regular with no murmurs. She has no cyanosis. Figure 17-1 shows her chest x-ray films.

-parapneumonic effusion

A 25-year-old man presents to your office complaining of a 3-month history of rhinorrhea, itchy eyes, and exertional cough and wheezing. These symptoms have been progressively worsening over the past few months. His past medical history is significant for seasonal allergies to pollen and ragweed. His family history is significant only for hypertension in both parents. His siblings and children are in good health without allergies or respiratory illness. His social history is negative for smoking. He has worked as an animal laboratory technician for the last 6 months. On questioning, his symptoms were initially more severe toward the end of the work week but are now continuous. He has been taking over-the-counter antihistamines, which helped initially but have not relieved his allergic symptoms. On review of systems, he has noted hives that are less prominent now that he has been taking the antihistamines on a regular basis. On examination, his body mass index (BMI) is 23, blood pressure is 120/75 mm Hg, pulse is 72 beats/min, and respiratory rate is 18 breaths/min. His conjunctiva are injected, there is mild clear ocular discharge, and his nasal turbinates are boggy without visible polyps. His lung examination reveals prolonged inspiratory-to-expiratory ratio and end-expiratory wheezing at the bilateral bases. His heart examination is unremarkable and there is no peripheral edema. What further testing should be done

-peak flow -allergy testing -methacholine challenge -spirometry -CXR -pulse ox -CBC with differential

What is the treatment for meningococcus for culture proven meningococcal meningitis

-penicillin G

The pain of pneumonia is usually described how

-pleuritic

What fungi commonly infects pts with AIDS and causes pulmonary symptoms

-pnemocystis jiroveci

What 4 things are linked to a reduction in the incidence of AOM

-pneumococcal and influenza vaccines -tabacco smoke avoidance -increased breast feeding

What are the 3 most common bacteria implicated in COPD exacerbations

-pneumococcus -H influenzae -moraxella catarrhalis

A 39-year-old homeless man presents to the emergency department (ED) with a nonproductive cough and subjective fever. He says that his illness has been worsening over the past 2 weeks, originally starting with dyspnea on exertion and now he is short of breath at rest. On questioning, he tells you that he lives in a homeless shelter when he can, but he frequently sleeps on the streets. He has used IV drugs (primarily heroin) "on and off" and has been sexually promiscuous with both men and women without barrier protection for many years. He denies any significant medical history, and only gets medical care when he comes to the ED for an illness or injury. On review of systems, he complains of chronic fatigue, weight loss, and diarrhea. On examination, he is a thin, disheveled man appearing much older than his stated age. His temperature is 100.4°F (38.0°C), his blood pressure is 100/50 mm Hg, his pulse is 105 beats/min, and his respiratory rate is 24 breaths/min. His initial oxygen saturation is 89% on room air, which comes up to 94% on 4 L of oxygen by nasal cannula. Significant findings on examination include dry mucous membranes, a tachycardic but regular cardiac rhythm, a soft and nontender abdomen, and generally wasted-appearing extremities. His pulmonary examination is significant for tachypnea and fine crackles bilaterally, but there are no visible signs of cyanosis on extremities. His chest x-ray reveals diffuse, bilateral, interstitial infiltrates that look like "ground glass." What is most likely the cause of the pts current pulmonary complaints? What underlying illness does this pt most likely have?

-pneumocystis jerovichi -AIDS

What can be used to treat post herpetic neuralgia in pts with shingles

-pregabalin or gabapentin

Which pts should you not give the varicella vaccine to since it is a live vaccine

-pregnant pts -immunocompromised pts

What are the 3 stages of syphilis

-primary: *painless ulcer* at the site of infection -secondary: skin rash that will spread to palms and soles, neuro symptoms, or ophthalmologic abnormalities, *condyloma lata* -tertiar: *granulomatous lesions (gummas)

If an individual dies from a PE, what is the usual cause of death

-progressive right heart failure

What are indications for Cl sweat testing

-prolapsed retum -pancreatitis -chronic diarrhea -steatorrhea -meconium ileus

What are the SE of fluroquinolones like Levofloxacin

-prolonged QT -increased risk of C diff -tendonopthy/rupture -altered mental status in older individuals

In a parapneumonic effusion, the _____ and ____ will be high and the ___ will be low if it is caused by a bacterial exudate

-protein and LDH -glucose

What are the symptoms of horner syndrome What is typically the caue

-ptosis -loss of pupillary dilation (miosis), -loss of sweating on the ipsilateral side (anhidrosis) caused by compression of the superior cervical ganglion - a superior sulcus tumor.

How do you screen for latent TB infection

-purified protein derivative (PPD) skin testing -Quantiferon TB Gold assay and the T SPOT TB assay

If a child has AOM, what will you see on PE

-red bulging tM with middle ear effusion and decreased mobility but either pnuumatic otoscopy and or tympanometry

What is Milliary TB? Adrenal involment is common in milliary TB and may cause what?

-refers to hematogenously disseminated tuberculosis, and describes the radiographic or pathologic finding of 1- to 2-mm granulomas that resemble millet seeds (hence the name). -Adrenal involvement is common in military TB, and may cause adrenal insufficiency.

Sputum that is what color is more likely to be a bacterial penumonia

-rust, bloody, brown, or green

What is the worst type of PE and why

-saddle embolus -it blocks both lungs

what organism causes typhoid fever

-salmonella enterica

D dimer is a highly __________ test, but it not very ________

-sensitive -specific

A 52-year-old man presents because of coughing and SOB. He has had multiple office visits in the past few years for similar reasons. He has a chronic "smoker's cough," but reports that in the past 2 days his cough has increased, his sputum has changed from white to green in color, and he has had to increase the frequency with which he uses his albuterol inhaler. He denies having a fever, CP, peripheral edema, or other symptoms. His medical history is significant for hypertension, peripheral vascular disease, and two hospitalizations for pneumonia in the past 5 years. He has a 60-pack-year history of smoking and continues to smoke two packs of cigarettes a day. On examination, he is in moderate respiratory distress. His temperature is 98.4°F, his blood pressure is 152/95 mm Hg, his pulse is 98 beats/min, his respiratory rate is 24 breaths/min, and he has an oxygen saturation of 94% on room air. His lung examination is significant for diffuse expiratory wheezing and a prolonged expiratory phase of respiration. There are no signs of cyanosis. The remainder of his examination is normal. A chest x-ray done in your office shows an increased anteroposterior (AP) diameter and a flattened diaphragm, but otherwise he has clear lung fields What interventions would be most helpful to reduce the risk of future exacerbations of this condition?

-smoking cessation -influenza, pneumococcal, and pertussis vaccines -medications based on how severe the COPD is

What type of lung cancer is the most likely to cavitate, produce PTH, and present with hypercalcemia

-squamous cell

What are the 3 types of non small cell lung cancers

-squamous cell -adenocarcinoma -large cell

What are the common bacterial pathogens for acute otitis media

-strep pneumoniae -H influenzae -moraxella catarrhalis

What is the treatment for hand foot and mouth disease

-supportive care such as rest and tylenol

You see a pt with otitis externa who has an erythematous pinna. How should they be treated

-systemic oral ciprofloxacin

What are the main criteria for tPA administration

-systolic BP <90 in the absence of absolute contraindications of tPA

What signs would you look for on exam or history if you expected a PE

-tachycardia -unilateral swelling of LE -cords -Homans -recent long travel -recent surgery -oral contraceptives -hormones -smoking -family history (factor 5 liden defect) -low pulse ox

What is the most common symptoms of a PE? What are other symptoms

-tachypnea -tachycardia -mild fever -pleuritic CP -dyspnea -increased RR -DVT -hypoxemia -diaphoresis -elevated JVP

What physical exam findings should be taken into consideration when using the the pneumonia severity index (PSI/PROT score)

-tachypnea -fever -hypotension -tachycardia -altered mental status

A 32-year-old woman presents to the emergency center complaining of productive cough, fever, and chest pain for 4 days. She was seen 2 days ago in her primary care physician's clinic with the same complaints, was diagnosed clinically with pneumonia, and was sent home with oral azithromycin. Since then, her cough has diminished in quantity. However, the fever has not abated, and she still experiences left-sided chest pain, which is worse when she coughs or takes a deep breath. In addition, she has started to feel short of breath when she walks around the house. She has no other medical history. She does not smoke and has no history of occupational exposure. She has not traveled outside of the United States and has no sick contacts. On physical examination, her temperature is 103.4°F, heart rate is 116 bpm, blood pressure is 128/69 mm Hg, respiratory rate is 24 bpm and is shallow. Her pulse oximetry is 94% saturation on room air. Physical examination is significant for decreased breath sounds in the lower half of the left lung fields posteriorly, with dullness to percussion about halfway up. There are a few inspiratory crackles in the mid-lung fields, and her right side is clear to auscultation. Her heart is tachycardic but regular with no murmurs. She has no cyanosis. Figure 17-1 shows her chest x-ray films. What is your next step

-thoracentesis to tap the fluid -LDH -glucose -cell count/culture -hematocrit -pH -cytology

Why is pyridoxine frequently added to the regimen of TB treatment

-to prevent peripheral neuropathy caused by IND due to a B6 deficienty

If the fluid in a pleural effusion is clear and straw colored, is it exudative or transudative

-transudative

MRSA cause penumonia requires treatment with what medication

-vancomycin

The typical pneumonia organisms are more common in what pts?

-very young and older pts

What are the symptoms of hand food and mouth disease

-vesicular enanthem on tongue, lips, in mouth -maculovesicular rash on hands, feet, and buttocks and groin

What are the symptoms of shingles

-vesicular eruption usually along a single dermatome that does not cross the midline -postherpetic neuralgia

What is see on a CXR in PE

-wedge shape

When would you impliment myringotomy with PE tubes

-when fluid does not resolve -recurrent AOM -hearing loss is noted

What is defined as recurrent AOM

-≥3 in the previous 6 months or ≥4 in the previous year with 1 in the previous 6 months

What are the 4 stages of HIV/AIDs infection

1. No AIDs defining illnes and either a CD4 count > or equal to 500 or % of total lymphocytes >29 2. No AIDs defining illness and either CD4 count of 200 to 499 or % between 14-28 3. CD4 count <200 or % <14 and documentation of AIDS defining condition 4. Unknown laboratory parameters with AIDS defining condition

asthma is diagnosed in about ______% of children adn ______% of adults with increasing prevalence in economically developed countries

10, 5

Oxygen is the only intervention that has been shown to decrease mortality in COPD pts if it is used for at least _______ hours a day

15

What is the recomended treatment for children with AOM

80 to 90 mg/kg/d for 7-10 days amoxicillin

An obese 50-year-old man with a history of asthma returns with complaints of occasional dyspepsia and nocturnal cough. He wakes up in the morning with a sour taste in his mouth. His current medications include inhaled corticosteroid and a short-acting β2-agonist. Which of the following should be your next step? a.24-Hour esophageal pH monitoring b.Chest radiograph c.Initiation of omeprazole d.Short course of oral corticosteroids e.Initiation of allergy desensitization

Answer: C The dyspepsia and the sour taste suggest GERD. Aside from acid suppression, other recommendations include dietary modifications and weight reduction. 24-Hour esophageal pH monitoring is indicated only if there is no response to treatment.

All pts with suspected pneumonia should have what imaging done

CXR

1. A 76-year-old widowed man who lives alone presents to clinic with increasing shortness of breath and chest pain at rest for the past 2 weeks. He has had chronic hypertension and coronary artery disease (CAD) for 20 years for which he takes hydrochlorothiazide (HCTZ), enalapril, and aspirin 81 mg daily. Other medical problems include hyperlipidemia, peripheral vascular disease, and gastroesophageal reflux disease (GERD) which are controlled by lovastatin, warfarin, and omeprazole. Two years ago, he suffered a cerebrovascular accident that was localized to the brain stem. He now has dysphagia and is noted to cough frequently at night. He has no cough at present and has not been able to take his temperature at home. Which of the following is the best next step? A Upper endoscopy B Removal of angiotensin-converting enzyme (ACE) inhibitor C Nitroglycerine patch D Chest radiograph

D Chest radiograph This patient most likely has aspiration pneumonia. With impairment of the gag reflex after cerebrovascular accident (CVA), he is more likely to aspirate during sleep, indicated by his cough. His GERD is well controlled by medication, so upper endoscopy is not warranted at this time. Nitroglycerine patches may be indicated if he described symptoms more related to angina. An ACE inhibitor would cause a cough unrelated to the time of day.

1. A 17-year-old adolescent boy presents to the ER with a temperature of 101.0°F (38.3°C), a deep nonproductive cough, and generalized malaise for 3 days. He doesn't recall being around any particular sick contacts but is around many people in his after-school job in sales and at school. He states that he never had the chicken pox and is unaware of what immunizations he received as a child. He was diagnosed at age 12 with leukemia but has since been healthy. He is worried that his cancer may no longer be in remission. A chest x-ray reveals bilateral, diffuse infiltrates. Which of the following is the most likely cause of illness? A Pneumonia caused by S pneumoniae B Pneumonia caused by P jiroveci C Pneumonia caused by L pneumophila D Pneumonia caused by M pneumoniae E Pneumonia caused by H influenzae

D Pneumonia caused by M pneumoniae Bilateral, diffuse infiltrates are more likely to be seen in patients with pneumonia caused by atypical agents, such as Mycoplasma, than in patients with typical pneumonia or aspiration pneumonia. Legionella, another atypical pneumonia, usually is in older patients and the patient did not have diarrhea. It is more likely that the patient contracted an atypical pneumonia than having a relapse of leukemia with such profound immunodeficiency as to have contracted a Pneumocystis infection, with no prior symptoms.

1. A 35-year-old morbidly obese woman returns to clinic with sudden onset of night sweats, chills, shortness of breath, and cough productive of yellowish-green sputum. Her vital signs show a temperature of 104.0°F, with a respiratory rate of 30 breaths/min, heart rate of 100 beats/min, pulse oximetry is 93% on room air. She was seen 8 days ago for headache, fever of 102.0°F, nonproductive cough, and myalgias. She was prescribed a dose of oseltamivir for 5 days. She felt better after taking the medication initially but now feels she is getting worse. She is sent to the emergency room for expedited evaluation. Assuming admission for pneumonia, which of the following is the best empiric antibiotic treatment for this patient? A A 14-day trial of oseltamivir B Azithromycin C Penicillin D Levofloxacin E Ceftriaxone with vancomycin

E Ceftriaxone with vancomycin This patient is most likely suffering from postinfluenza pneumonia. Due to the higher risk of mortality associated with morbid obesity and some vital signs indicating risk of sepsis, this patient should be evaluated quickly for possible admission. If admitted for pneumonia, antibiotic coverage should cover for Pneumococcus and S aureus. Levofloxacin would be reasonable for community-acquired pneumonia but does not provide good coverage for staph infections.

T or F: LABA can be used as a rescue therapy

F

T or F: TB does not need to be reported to the local health department

F

T or F: people with CF are typically fertile

F

T or F: squamous cell carcinoma usually metastasizes early

F

T or F: the pulse ox will always be low in a pt having a PE

F

T or F: trichomoniasis if a non curable STI

F

T or F: anticoagulation dissolves existing clots

F -It allows for endothelialization and organization

T or F: If you get a pleural effusion with lung cancer, it is not necessarily considered metastatic

F It is considered metastatic

T or F: TB lymphadenitis is common in HIV pts, kids, and nonwhite women and is generally painful adenopathy

F It is usually painless

T or F: cultures have high sensitivity

F They have low sensitivity which means there are many false negatives, but a positive culture can help to guide treatment

T or F: bacterial sinusitis is more likely BL and will present with a persistent/high fever or symptoms that are worsening

F bacterial sinusitis is more likely UNILATERAL and will present with a persistent/high fever or symptoms that are worsening

T or F: PPD testing can be used to diagnose active infection

F due to the frequent false negative results

T or F: Otitis media with effusion requires abx

F, but hearing should be monitored as they can cause other problems

During spirometry, what 3 main values are measured

FEV1, FVC and the FEV1/FVC ratio

What are healed lesions from TB called

Ghon lesions

What is the most commonly sexually transmitted infection

HPV

What is the triad of CF

It is characterized by the triad of chronic obstructive pulmonary disease, pancreatic exocrine deficiency, and abnormally high sweat electrolyte concentrations.

What are the light criteria

Must meet at least one of the following criteria 1.Pleural fluid protein/serum protein ratio >0.5 2.Pleural fluid LDH/serum LDH ratio >0.6 3.Pleural fluid LDH >2/3 the upper limit of normal for serum LDH

A 48-year-old woman calls 911 and is brought to the emergency center complaining of a sudden onset of dyspnea. She reports she was standing in the kitchen making dinner, when she suddenly felt as if she could not get enough air, her heart started racing, and she became light-headed and felt as if she would faint. She denied chest pain or cough. Her medical history is significant only for gallstones, for which she underwent a cholecystectomy 2 weeks previously. The procedure was complicated by a wound infection, requiring her to stay in the hospital for 8 days. She takes no medications regularly, and only takes acetaminophen as needed for pain at her abdominal incision site. On examination, she is tachypneic with a respiratory rate of 28 bpm, oxygen saturations 84% on room air, heart rate 124 bpm, and blood pressure 118/89 mm Hg. She appears uncomfortable, diaphoretic, and frightened. Her oral mucosa is slightly cyanotic, her jugular venous pressure is elevated, and her chest is clear to auscultation. Her heart rhythm is tachycardic but regular with a loud second sound in the pulmonic area, but no gallop or murmur. Her abdominal examination is benign, with a clean incision site without signs of infection. Her right leg is moderately swollen from mid-thigh to her feet, and her thigh and calf are mildly tender to palpation. Laboratory studies including cardiac enzymes are normal; her electrocardiogram (ECG) reveals only sinus tachycardia, and her chest x-ray is interpreted as normal.Top of Form What is the most likely diagnosis

PE

What ECG finding is associated with PE

S1Q3T3

T or F: HIV infection is reportable to local health authorities, but partner notification laws vary state by state

T

T or F: If you cannot see the TM on exam, you should not try to flush out the ear canal in pts with otitis externa due to a risk of perf

T

T or F: LABAs have been associated with increased mortality

T

T or F: Pleural LDH correlates with the degree of pleural inflammation and, along with fluid protein, should always be sent in the initial evaluation.

T

T or F: Systemic steroids shorten the course of the exacerbation and may reduce the risk of relapse in pts with COPD

T

T or F: a negative sweat cl test does not exclude CF

T

T or F: adenocarcinomas metastasize early

T

T or F: if you try treating chicken pox with aspirin, you increase the risk of developing rye syndrome

T

T or F: in children older than 6 months with mild symptoms, watchful waiting period of a few days may be indicated since many AOM cases self resolve

T

T or F: middle ear fluid can persist for up to several months after AOM

T

T or F: most infections with HPV are asymptomatic and are cleared by the bodies immune system

T

T or F: oIf the individual has been on steroids for a few days, their WBC count may be artificially increased by the steroids

T

T or F: popcorn and bull's eye calcifications suggest a benign process, whereas absence of calcification increases the likelihood of malignancy

T

T or F: pts who have had a conventional surgical embolectomy have a higher mortality (50%) if the procedure is done emergently

T

T or F: smoking cessation does not result in significant improvement in pulmonary function, but it does reduce the rate of further deterioration to that of a nonsmoker

T

T or F: the duration of treatment relates to the risk of recurrence of PE

T

T or F: the lower the risk of the pt, the higher the PPD induration has to be. The higher the risk of the pt, the lower the PPD induration has to be

T

How do you differentiate between intermitent and persistent asthma

THE RULE OF 2 -2 or fewer symptoms or SAVA use a week -2 or less nocturnal awakenings a month -2 or less episodes needing for oral steroids in a year

A 30-year-old man comes to your office complaining of a swollen neck. He noticed it 10 days ago when he was stung by a bee on the right side of his anterior neck. The area has continued to enlarge. It is no longer tender. It was erythematous after the sting, but the redness has resolved. He notes no other symptoms. On exam, you find a 2-cm firm, somewhat tender, enlarged lymph node in the right anterior cervical chain. The node is mobile, nonfluctuant, with no surrounding erythema. There are also shotty anterior and posterior cervical nodes in addition to the larger node described. You find neither supraclavicular, axillary, nor inguinal lymphadenopathy, nor splenomegaly. What elements of the presentation make malignancy LESS likely? A) The node is freely mobile. B) The node is only 2 cm. C) The node is associated with trauma (bee sting). D) The node is tender. E) All of these help to rule out malignancy.

The correct answer is "A." Nonmalignant nodes are generally less than 1 cm in size, freely mobile, and rubbery in constancy. Malignant nodes tend to be larger, rock-hard, and fixed. They become immobile secondary to tumor invasion into the surrounding tissues and/or inflammation. Remember that pain in a node is not always indicative of an inflammatory or benign process. Hemorrhage into, or necrosis of, a malignant node can cause capsular distention leading to pain. "C" is of particular note. This patient's bee sting was quite a while ago and is unlikely to be a useful part of the history unless there is ongoing inflammation. Patients will often attribute a physical malady to something in their lives whether or not it makes sense from a biological and medical perspective (one of us had a patient who swore that his purulent sputum was because his lungs were connected to his gallbladder). Of course, lymphadenopathy is not the only source of neck masses. See Table 20-3 for the differential diagnosis of neck masses in adults.

A 2-year-old is brought to your office by her mother who is concerned that she has been pulling at her left ear since late last night and has a fever of 101.3°F. She has had recurrent bouts of these symptoms, the last of which was 9 months ago. Each time, the symptoms resolved with one "shot." She is alert and interactive. She has some evidence of mucoid discharge from her nares bilaterally. Which of the following is NOT a criterion for tympanostomy tubes? A) Chronic bilateral effusions for more than 3 months with unilateral hearing loss. B) Failure of antibiotic therapy to prevent recurrent otitis media. C) Language delay secondary to otitis media. D) Greater than 20 dB hearing loss bilaterally.

The correct answer is "A." Patients should meet the criteria listed above before being considered for tympanostomy tubes. Note that this requires that patients also meet the criteria for prophylactic antibiotic therapy (≥3 episodes of AOM in 6 months or ≥4 episodes in 12 months). A modification of "A" is also a criterion: chronic bilateral effusions for more than 3 months with bilateral hearing loss. Although included as a criterion, there is no evidence that tympanostomy tubes improve language development in the short or long term. If fluid persists in the middle ear after AOM, it is termed OME.

You treat a patient with otitis externa with neomycin/polymyxin drops, and the symptoms persist and possibly worsen a little after 5 days. The patient has no fever and no signs of cellulitis around the ear. Of the following possibilities, which is the LEAST likely to explain her persistent symptoms? A) Resistant organisms. B) Noncompliance with medical recommendations. C) Misdiagnosis of otomycosis. D) Development of contact dermatitis.

The correct answer is "A." Several things could explain her persistent symptoms. First, the patient should be questioned regarding compliance. Is she still swimming despite advice to the contrary? Is she using the drops at least TID and letting them soak into her ear? Another issue is the development of an allergic reaction, especially in response to neomycin (up to 35% of patients treated chronically with topical neomycin develop a dermatitis). Patients may also "fail" treatment for otitis externa due to misdiagnosis. Otomycosis, a fungal infection of the auditory canal, causes redness, discharge, itching, and sometimes pain. With respect to option "A," treatment failures due to antibiotic resistance are uncommon, and as noted above, antibiotics are not even necessary in the treatment of most cases of otitis externa.

A 42-year-old businesswoman presents to your office with the chief complaint of 2 days of headache, sore throat, and nasal congestion productive of green mucus. She denies any fever, contact with ill persons, and gastrointestinal symptoms, but she does have a history of seasonal allergies. On exam, she has completely normal vital signs. Her posterior oropharynx has mild erythema and postnasal drainage but no exudates. There is nasal mucosal erythema and swelling with clear rhinorrhea. Her neck is supple with no adenopathy. Respirations are clear. The most likely agent causing her symptoms and the most common cause of acute rhinosinusitis is: A) Rhinovirus. B) S. pneumoniae. C) H. influenzae. D) M. catarrhalis. E) Norwalk virus.

The correct answer is "A." Viruses are the most common cause of URIs (or colds or rhinosinusitis). Up to 50% of colds are caused by the 100 different serotypes of rhinoviruses. Other viruses that commonly cause colds include coronaviruses, RSV, parainfluenza, and influenza. Norwalk virus typically causes an intestinal illness. The bacteria listed ("B"-"D") are also associated with infections of the upper respiratory tract, particularly otitis, and sinusitis, but are much less common than viruses.

A 42-year-old businesswoman presents to your office with the chief complaint of 2 days of headache, sore throat, and nasal congestion productive of green mucus. She denies any fever, contact with ill persons, and gastrointestinal symptoms, but she does have a history of seasonal allergies. On exam, she has completely normal vital signs. Her posterior oropharynx has mild erythema and postnasal drainage but no exudates. There is nasal mucosal erythema and swelling with clear rhinorrhea. Her neck is supple with no adenopathy. Respirations are clear.The patient is initially convinced that only antibiotics will make her better. Through skillful negotiation (and a hefty dose of haloperidol), you manage to avoid prescribing antibiotics for what you strongly suspect is a viral infection. Two weeks later, the patient returns. Initially, she improved, but then she developed subjective fever, face pressure, maxillary tooth pain, and copious green nasal drainage. You now suspect that a bacterial sinusitis has developed. The look on her face says, "I told you so...and I am not taking any more of that haloperidol stuff." She has no drug allergies. Which of the following treatments do you offer as first-line therapy? A) Azithromycin. B) Trimethoprim/sulfamethoxazole. C) Prednisone. D) Ceftriaxone. E) None of the above.

The correct answer is "B." Most guidelines addressing the treatment of acute bacterial sinusitis recommend narrow-spectrum antibiotics, such as amoxicillin, trimethoprim/sulfamethoxazole, and doxycycline. "C" is incorrect. There is no reason to prescribe steroids here. "D," ceftriaxone, might be considered in cases of treatment failure, but oral antibiotics are generally preferred. Of course, you should continue to recommend that the patient use decongestants and other symptom-oriented therapies. -FARLEY "• Ceftriaxone is IM or IV and is overkill. Prednisone wont treat the infection. There is a lot of resistance to azithromycin. Trimethoprim has poor coverage. The answer for the above question should be high dose amoxicillin or augmentin"

A 30-year-old man comes to your office complaining of a swollen neck. He noticed it 10 days ago when he was stung by a bee on the right side of his anterior neck. The area has continued to enlarge. It is no longer tender. It was erythematous after the sting, but the redness has resolved. He notes no other symptoms. On exam, you find a 2-cm firm, somewhat tender, enlarged lymph node in the right anterior cervical chain. The node is mobile, nonfluctuant, with no surrounding erythema. There are also shotty anterior and posterior cervical nodes in addition to the larger node described. You find neither supraclavicular, axillary, nor inguinal lymphadenopathy, nor splenomegaly. What is the most appropriate next step in the management of this patient? A) Empiric antibiotics. B) Observation for 4 weeks. C) Open biopsy of the node. D) Fine needle aspiration of the node. E) Incision and drainage.

The correct answer is "B." Patients with lymphadenopathy can be observed for 3-4 weeks unless there is a suggestion of malignancy (e.g., fever, night sweats, and weight loss). Note that a 3-4 week delay makes no difference in patient outcome if the node does turn out to be malignant. If adenopathy does not resolve, further evaluation including biopsy can be done. Open biopsy and fine needle aspiration each have advantages and disadvantages, but either could be used to obtain tissue. "A," empiric therapy with antibiotics, is possibly correct if you suspect a lymphadenitis or a bacterial infection causing secondary lymphadenopathy. However, in our patient, there is no tenderness or other signs of infection, arguing against lymphadenitis. •Antibiotics would not be necessary bc its not red, tender or fluctinuate. That is also why you would not biopsy or drain yet. C or D would be more likely if it did not go away in a month

A 42-year-old businesswoman presents to your office with the chief complaint of 2 days of headache, sore throat, and nasal congestion productive of green mucus. She denies any fever, contact with ill persons, and gastrointestinal symptoms, but she does have a history of seasonal allergies. On exam, she has completely normal vital signs. Her posterior oropharynx has mild erythema and postnasal drainage but no exudates. There is nasal mucosal erythema and swelling with clear rhinorrhea. Her neck is supple with no adenopathy. Respirations are clear. Which of the following does NOT increase the likelihood that a patient has a bacterial sinusitis? A) Persistence of symptoms for greater than 7 days. B) Thickened nasal mucosa or effusion on CT scan. C) Maxillary tooth pain. D) Unilateral maxillary sinus pain.

The correct answer is "B." Radiography is particularly poor at diagnosing bacterial sinusitis. Thickened nasal mucosa is only 40-50% specific for sinusitis (flip a coin, it's cheaper). The other problem with imaging is that essentially all patients with a URI will have fluid in the sinuses. Clinical criteria are more helpful in predicting the presence of bacterial sinusitis; thus, "A," "C," and "D" are correct. Additionally, a biphasic course, sometimes referred to as "double sickening," is a good predictor of bacterial sinusitis: if the patient initially improves and then gets worse, consider a secondary infection. Thick, green, nasal drainage deserves special mention because green nasal drainage does not necessarily mean that bacteria are present. Secretions will turn green with a viral illness, as with anything else that concentrates protein in the mucous (e.g., anticholinergics).

If this patient is found to have laryngeal cancer, what pathologic variant is most likely? A) Adenocarcinoma. B) Squamous cell carcinoma. C) Schneiderian papilloma. D) Neuroblastoma.

The correct answer is "B." Upon pathologic examination, the great majority of head and neck cancers are found to be squamous cell carcinomas. Adenocarcinoma may arise from the gastrointestinal tract and could be seen on laryngoscopy but would rarely occur on the vocal cords. Schneiderian papillomas ("C") are polyps that arise from the nasal and sinus mucosae, are associated with HPV, and may transform into carcinomas. Neuroblastomas ("D"), which arise from the sympathetic nervous system, rarely occur in the head and neck region.

you want to remind him of risk factors for laryngeal cancer, which include all of the following EXCEPT: A) Tobacco smoking. B) Alcohol use. C) Epstein-Barr virus (EBV). D) Family history of head and neck cancers. E) Male sex

The correct answer is "C." EBV infection is associated with the development of nasopharyngeal cancer, not laryngeal cancer. Additionally, EBV infection has been associated with Burkitt lymphoma (children in Africa), Hodgkin disease, and non-Hodgkin lymphoma. Tobacco and alcohol use are independent risk factors for the development of most types of head and neck cancers (oral, laryngeal, etc.), and the two substances may act synergistically in the promotion of these cancers. A family history of head and neck cancer has a weaker association, but the association is still present. Males are two to four times more likely to have head and neck cancers compared with females. -EBV causes mono

A 30-year-old man comes to your office complaining of a swollen neck. He noticed it 10 days ago when he was stung by a bee on the right side of his anterior neck. The area has continued to enlarge. It is no longer tender. It was erythematous after the sting, but the redness has resolved. He notes no other symptoms. On exam, you find a 2-cm firm, somewhat tender, enlarged lymph node in the right anterior cervical chain. The node is mobile, nonfluctuant, with no surrounding erythema. There are also shotty anterior and posterior cervical nodes in addition to the larger node described. You find neither supraclavicular, axillary, nor inguinal lymphadenopathy, nor splenomegaly. Which of the following tests is NOT helpful in arriving at a diagnosis in a patient with GENERALIZED lymphadenopathy? A) CBC. B) Chest radiograph. C) Glucose, BUN, creatinine. D) HIV. E) Heterophile antibody.

The correct answer is "C." Glucose, BUN, and creatinine are not likely to help you with the diagnosis of generalized lymphadenopathy. Lymphadenopathy in primary care is malignant approximately 1% of the time. After a period of observation, the workup should proceed in stages. First step: CBC, chest radiograph. Second step: PPD, HIV, RPR, ANA, heterophile antibody. Final step: biopsy. Use your clinical judgment to determine the extent of testing necessary in any individual case. •CBC can tell you if there is evidence of infection with a WBC count, esp if you do a differential •CXR can help r/o lung malignancies, TB, HIV signs, etc •HIV can be a sign of generalized lymphadenopathy •Heterophile antibody is a test for mono

During your examination of the oropharynx of a pt suspected to have laryngeal cancer, you also encountered a small, white, indurated plaque on the underside of the tongue. When you scraped the plaque with a tongue blade, nothing happened. This lesion is most appropriately described as: A) Squamous cell carcinoma. B) Candida albicans. C) Leukoplakia. D) Geographic tongue. E) Aphthous ulcer.

The correct answer is "C." Leukoplakia is a premalignant lesion of the oropharynx (about 5% will progress to cancer over 10 years). It occurs in response to trauma and/or exposure to irritants and carcinogens, having an especially strong association with smokeless tobacco (e.g., "snuff," "chew") use. In fact, the lesion could be squamous cell carcinoma ("A"), and it should be biopsied. However, it would be premature to diagnose the patient with squamous cell carcinoma, and the lesion is more accurately described as leukoplakia. "B," C. albicans lesions, may look just like leukoplakia (white plaques on oropharyngeal mucosa), but you should be able to scrape some of the plaques off with a tongue blade (although thrush can be remarkably adherent). "D," geographic tongue, is so named because of the meandering white-bordered patches that occur on the dorsum of the tongue. It is most often asymptomatic, and the lesions vary in shape (or completely resolve) over time. Finally, "E," an aphthous ulcer, is just that—an ulcer, not a plaque. You should not confuse leukoplakia for an aphthous ulcer. •Thrush caused by candida albicans can be scraped off and you are left with beefy red color •Geographic tongue is on the top and it looks like a map and the pattern changes

You are treating a pt for otitis externa but she did not have success with the initial treatment of polymyxin/neomycin drops. On exam, you notice fine, white, cotton-like fibers filling the ear canal along with the other debris. The exam is otherwise unchanged. What is your next step? A) Admit for intravenous (IV) antibiotic and antifungal therapy. B) Clean the ear canal under direct otoscopy and add oral amoxicillin/clavulanate to her medication regimen. C) Clean the ear canal under direct otoscopy and add topical clotrimazole 1% to her medication regimen. D) Order CT or MRI of the head and neck to rule out abscess. E) Refer to an otolaryngologist.

The correct answer is "C." On exam, you have identified signs of otomycosis. It could be that the otomycosis was present initially or developed in the interval with antibiotic administration. Otomycosis is usually due to Aspergillosis; Candida only represents 10-20% of cases. Thorough cleaning of the ear canal is an important part of therapy. A topical antifungal that is active against Aspergillosis is recommended (e.g., clotrimazole, miconazole, and nystatin).

a. When treating AOM, which of these individuals should be considered a treatment failure and switched to another antibiotic? A) A patient with a fever that continues at 24 hours after starting an oral antibiotic. B) A child who is still tugging at his ear 5 days into a course of antibiotics. C) A symptomatic child who still has a bulging, red, immobile TM 3 days after starting antibiotics. D) A child who continues to have rhinorrhea 1 week after starting antibiotics. E) All of the above.

The correct answer is "C." You should consider switching to a different antibiotic in patients who remain symptomatic at 3 days and who continue to have positive findings on pneumatic otoscopy. Symptoms are not enough: they are unreliable. Remember, since most of these are viral infections, you are not doing a whole lot of good with your antibiotics anyway. "A" is incorrect because 24 hours is not sufficient to determine if a particular antibiotic will be effective. "B" is incorrect because patients pull at their ears for a number of reasons besides AOM (such as "Ha! I just discovered I have ears!"). "D" is incorrect. If you chose this one, back to Microbiology 101 for you! Rhinorrhea does not respond to antibiotics and is most likely not bacterial in origin.She returns with persistent pain and fever after taking amoxicillin for 3 days. On exam, you find evidence of persistent AOM. You switch the patient to your favorite second-line antibiotic. You see her back in 2 weeks for an ear check and find complete resolution. The mother asks what she could do to avoid these troublesome infections in the future.

a. All of the following have been shown to reduce the incidence of recurrent otitis media EXCEPT: A) Antibiotic prophylaxis. B) Conjugate pneumococcal vaccine and/or influenza vaccine. C) Tympanostomy tubes. D) Tonsillectomy.

The correct answer is "D." Primary tonsillectomy has not been shown to reduce the recurrence of otitis media. However, adenoidectomy with or without tonsillectomy will reduce the rate of recurrent otitis media in patients who already have tympanostomy tubes. "A," the use of antibiotic prophylaxis, will reduce recurrent otitis media. Antibiotic prophylaxis should be considered in the patient who has had ≥3 episodes of otitis media in 6 months or ≥4 episodes in 12 months. Reasonable choices for antibiotics include amoxicillin and trimethoprim/sulfamethoxazole. Give half of the usual daily dose. This is generally given at bedtime. Often, antibiotics can be stopped during the summer since an upper respiratory infection (URI) is the precipitant of most cases of otitis media (remember that the great majority are solely viral). Pneumococcal vaccine (e.g., Prevnar) will reduce the risk of recurrence in children with severe and recurrent AOM. The same is true of influenza vaccine. Also recommended to reduce the frequency of AOM: avoid pacifier use, avoid bottle propping at night, avoid smoke exposure, and encourage breast feeding for at least the first 6 months of life.The patient returns 4 weeks later with the mother saying, "She is still pulling at her left ear." There are no other complaints. On exam, you find the left TM is without redness or opacity but there is still a fluid level. The right ear exam is unremarkable.

Each of the following findings is diagnostic of acute otitis media (AOM) EXCEPT: A) Profuse, purulent ear discharge without other evidence of otitis externa. B) Air-fluid level behind the tympanic membrane (TM) with marked redness of the TM and poor movement with pneumatic otoscopy. C) Bulging, thickened yellow and red TM that does not move well with pneumatic otoscopy. D) Bubbles in fluid behind the TM with impaired mobility of the TM on pneumatic otoscopy. E) Yellow, opaque TM, poor movement with pneumatic otoscopy, and substantial ear pain.

The correct answer is "D." Suspected ear infections drive many parents to bring their children to a family physician. There may be fluid in the middle ear that is not infected (otitis media with effusion [OME]). In order to diagnose AOM, you need evidence of fluid in the middle ear and inflammation. Middle ear effusion is diagnosed by (1) bubbles and/or air-fluid level behind the TM; (2) two or more of the following: decreased or absent TM movement with pneumatic otoscopy, opacification of the TM, and discoloration of the TM (yellow, white, blue). These findings get you to OME but not AOM. To diagnose AOM, you will need to have OME with evidence of acute inflammation, such as marked pain, thickened and/or bulging TM, and reddened TM. For these reasons, "B," "C," and "E" are examples of AOM. "A," purulent otorrhea without evidence of otitis externa, would be the one exception where you can diagnose AOM without even seeing the TM. "D" describes OME without inflammation.Your patient's left TM is opaque, red, and immobile upon pneumatic otoscopy.

a. Which of the following statements best characterizes the role of antibiotics in the treatment of AOM? A) Antibiotics have been shown to reduce suppurative complications of AOM, such as mastoiditis, in developed countries. B) The majority of patients with AOM benefit from the use of antibiotics. C) The use of antibiotics for AOM reduces hearing loss and benefits language development. D) With or without antibiotics, about 75% of children have resolution of AOM symptoms after 7 days. E) All of the above are true.

The correct answer is "D." The benefit of antibiotics for most children with AOM is marginal (number needed to treat [NNT] is about 12); thus, the option exists to observe and not even give antibiotics in children ≥2 years old who have mild symptoms, are not immunocompromised, and have good follow-up. The rest of the statements are incorrect. Antibiotics do not reduce suppurative complications in developedcountries, but they do seem to prevent suppurative complications in developing countries where sanitation and health-care access are not optimal. "B" is incorrect. The NNT with antibiotics is up to 12 in order to benefit 1 individual, and this benefit is limited to a 6% absolute reduction in those who have pain at days 2-7 (21% vs. 15%). "C" is incorrect as well, since treating with antibiotics does not impact these outcomes in any way. You prescribe amoxicillin for 10 days and suggest acetaminophen for comfort. A few days later, the patient's mother calls to say that she is no better. You ask her to come in to clinic for evaluation.

A 23-year-old female college student presents to your clinic complaining of ear pain. She is on the swimming team and notes that this pain occurs during swimming season. The pain is increased by motion of the pinnae. The external auditory canal is erythematous, edematous, and exquisitely tender when you try to use the otoscope to examine her TM. There is whitish debris in the external auditory canal. The most likely organism involved in this patient's disease is: A) Streptococcus. B) Haemophilus. C) Moraxella. D) Pseudomonas. E) Parainfluenza.

The correct answer is "D." This patient likely has otitis externa. The most common pathogenic organism isolated in cases of otitis externa is Pseudomonas followed closely by Staphylococcus aureus. However, up to one-third of cases of otitis externa are polymicrobial.

All of the following are potential causes of hoarseness EXCEPT: A) Vocal cord mass. B) Infectious laryngitis. C) Hypothyroidism. D) Lung malignancy. E) A vow of silence.

The correct answer is "E." Far from being a cause of hoarseness, voice rest is often recommended for patients with hoarseness due to overuse (e.g., singers). And nobody takes a vow of silence these days except for an occasional monk. All of the other options are known to cause hoarseness. Of particular note is "C": hypothyroidism can result in an accumulation of connective tissue elements, basically myxedema, in the vocal cords. Intrathoracic processes such as lung cancer can present with hoarseness (see "Helpful Tip" below).

Which of the following interventions has proven benefit in patients with OME? A) Oral decongestants. B) Oral antihistamines. C) Prolonged treatment (≥1 month) with oral antibiotics. D) Oral corticosteroids. E) None of the above.

The correct answer is "E." For persistent OME, there are no useful medical interventions. Autoinflation ("eustachian tube exercises" or forced exhalation with closed nose and mouth) is often recommended but has not shown a benefit (and try explaining how to do this to a 6-month-old). Patients may benefit from surgical intervention (see the question about tympanostomy tubes above).

A 25-year-old woman is seen in the clinic because her father, who recently immigrated from South America, was diagnosed with and has been treated for tuberculosis. She denies a cough and her chest radiograph is normal. A PPD test shows 10 mm of induration. Her only medication is an oral contraceptive. Which of the following is the best next step? A Isoniazid. B Combination therapy including isoniazid, rifampin, and pyrazinamide. C Observation. D Induce three sputum samples.

The correct answer is A. Explanation: Because this woman is a household contact of a patient with active TB, she is among the highest risk group: her skin test would be considered positive with 5-mm induration. She has latent TB infection and should be offered treatment to prevent reactivation TB later in life. INH is the treatment of choice for exposure prophylaxis.

A 25-year-old white woman who is in training for a competitive marathon complains of "hitting a wall" and "getting short of breath quicker than she should." She complains of coughing at the end of her training runs, and states that she may be expecting too much of herself. She does not smoke, has no significant family history, and no history of occupational or environmental exposures. Her physical findings including lung examination are unremarkable. Spirometry reveals normal values both pre- and post-albuterol treatment. What would be the most reasonable first step in treatment of this patient? A Trial of albuterol MDI before exercise B Chest radiograph C Chest CT D Counseling for athletic burnout or stress E An echocardiogram (ECG) to rule out pulmonary hypertension or cardiac disorder

The correct answer is A. Explanation: Exercise-induced asthma or bronchoconstriction is a common, underdiagnosed condition in athletes. Many of the athletes are unaware of the problem. It is defined as a 10% lowering of forced expiratory volume in 1 second (FEV1) when challenged with exercise. It is much more common in high-ventilation sports and in cold, dry air. The incidence among cross-country skiers is as high as 50%. A physical examination and spirometry at rest will be normal unless there is underlying asthma. Methacholine challenge testing can be ordered, but if it is not available, a trial with an albuterol inhaler is reasonable. Pulmonary or cardiac dysfunction not found during the physical examination is much less likely and, therefore, an ECG and chest x-ray would not be indicated until common etiologies have been ruled out. Psychological causes are also a less likely etiology.

A 6-year-old girl is brought to your office by her mother because of a rash first noticed 1 week ago. Her mother reports that several children in her child's school have chicken pox but that her child has received all of her immunizations including two doses of the varicella vaccine. You observe the child actively playing with the toys in your waiting room before both the mother and child are brought back to the examination room. The child has a temperature of 100.4°F (38.0°C), a pulse of 90 beats/min, a blood pressure of 100/70 mm Hg, and a respiration rate of 20 breaths/min. The physical examination is unremarkable except for approximately 20 vesicles on erythematous bases sparsely scattered on the child's trunk and limbs. Which of the following is the most appropriate treatment? A. Supportive care B. Antiviral therapy C. Antibiotic therapy D. Immune globulin

The correct answer is A. You answered A. Explanation: The child has chicken pox caused by the varicella-zoster virus. While the child did receive two doses of the varicella vaccine and the vaccine is effective, sporadic breakthrough cases do occur. However, the cases are usually much less severe and have fewer complications than in unimmunized patients. Supportive care is advised, as this illness in this stage will be self-limited; after a week, antivirals likely have no benefit. Antibiotics and immune globulin have no role in the treatment of this patient.

A 24-year-old man has been treated with isoniazid, rifampin, and pyrazinamide for active pulmonary tuberculosis. After 3 months, he states that he is having numbness and tingling of both feet but no back pain. He denies taking other medications. Which of the following is the most appropriate next step? A Computed tomography (CT) scan of the lumbar spine. B Initiate pyridoxine. C Continue the tuberculosis agents and monitor for further neurologic problems. D Initiate a workup for tuberculosis adenopathy compression on the femoral nerve.

The correct answer is B. Explanation: Pyridoxine (vitamin B6) is important for preventing the peripheral neuropathy that can complicate isoniazid therapy. If the numbness were caused by Pott disease, he should have back pain and other neurologic findings, such as lower extremity weakness.

Three days after beginning oral amoxicillin therapy for otitis media (OM), a 4-year-old boy has continued fever, ear pain, and swelling with redness behind his ear. His ear lobe is pushed superiorly and laterally. He seems to be doing well otherwise. Which of the following is the most appropriate course of action? a. Change to oral amoxicillin-clavulanate b. Myringotomy and parenteral antibiotics c. Nuclear scan of the head d. Topical steroids e. Tympanocentesis

The correct answer is B. B. The child has mastoiditis, a clinical diagnosis that can require computed tomography scan confirmation. Treatment includes myringotomy, fluid culture, and parenteral antibiotics. Surgical drainage of the mastoid air cells may be needed if improvement is not seen in 24 to 48 hours

A 55-year-old man with congestive heart failure develops bilateral pleural effusions. Which of the following is the most likely pleural fluid characteristic if thoracentesis is performed? REFERING TO THE (RATIO IN PLEURAL FLUID TO BLOOD) A Pleural fluid LDH 39, LDH ratio 0.2, protein ratio 0.7 B Pleural fluid LDH 39, LDH ratio 0.2, protein ratio 0.1 C Pleural fluid LDH 599, LDH ratio 0.9, protein ratio 0.1 D Pleural fluid LDH 599, LDH ratio 0.9, protein ratio 0.7

The correct answer is B. Explanation: Congestive heart failure is commonly associated with bilateral pleural effusions, which are transudative, as a consequence of alteration of Starling forces. The effusions of heart failure are best managed by treating the heart failure, for example, with diuretics, and typically do not require thoracentesis. Per Light Criteria, all other options would be classified as an exudative pleural effusion.

A 33-year-old woman who is a nonsmoker has lost 30 lb and has a cough. She is noted to have a lung mass on chest radiograph. Which of the following lung cancers is the most likely cell type? A Squamous cell B Adenocarcinoma C Small cell D Large cell

The correct answer is B. Explanation: Eighty-five percent of patients with lung cancer of all histologic types have a smoking history. The most common form of lung cancer found in nonsmokers, young patients, and women is adenocarcinoma.

A 69-year-old man complains of gradually worsening dyspnea and a nagging cough over the past 3 months but no fevers. He is found to have a right-sided pleural effusion, which is tapped and is grossly bloody. Which of the following is the most likely diagnosis? A Parapneumonic effusion B Malignancy in the pleural space C Rupture of aortic dissection into the pleural space D Pulmonary embolism with pulmonary infarction Complete Quiz and View Results You will be able to view all answers at the end of your quiz.

The correct answer is B. Explanation: The most common causes of hemorrhagic pleural effusion are trauma, malignancy, and pulmonary embolism. Pulmonary embolism would be suggested by acute onset of dyspnea and pleuritic chest pain rather than this subacute presentation. Similarly, aortic rupture can produce a hemothorax but would have an acute presentation with pain and hemodynamic compromise.

A 45-year-old man has STI screening done at a screening fair at a local free clinic. He has never been tested for STIs before and is completely asymptomatic. He tests negative for HIV, gonorrhea, and Chlamydia but is notified that he has a positive RPR. What is the next appropriate step for him? A Treatment with penicillin G B FTA-ABS testing C Notification of his STI to the local health department D Repeat his STI panel as this is likely a false-positive test

The correct answer is B. Explanation: He has tested positive on his initial screening test for syphilis with a nontreponemal test. A confirmatory test with a treponemal test should be performed prior to making the diagnosis or implementing treatment. If he is confirmed as positive, he should then be treated with penicillin and notification should be made to the health department.

A 64-year-old woman comes to your office complaining of hoarse voice for 4 months. She has not had fever, sore throat, or a cough. On examination, she has expiratory wheezes in her left mid-lung fields. Which of the following is the best next step? A Prescribe antibiotics for bronchitis. B Order a chest x-ray. C Advise gargling with saltwater solution. D Prescribe an albuterol inhaler.

The correct answer is B. Explanation: This patient has chronic hoarseness and unilateral wheezing. This suggests an intrathoracic mass causing bronchial obstruction and impairment of the recurrent laryngeal nerve, causing vocal cord paralysis. Thus, an imaging study of the chest is essential.

A 1-year-old boy presents with fever, ear pain, and purulent discharge from both eyes. On examination, bilateral erythematous, bulging tympanic membranes (TMs), and purulent conjunctivitis are noted. Based on his symptoms and examination, amoxicillin-clavulanate is prescribed. The mother asks what she could do to prevent future ear infections. Which of the following is NOT one of the current recommendations to reduce the incidence of otitis media? a. Pneumococcal vaccine b. Influenza vaccine c. Xylitol d. Eliminating exposure to tobacco smoke e. Breast-feeding

The correct answer is C. C. Although some research shows xylitol use can reduce AOM, compliance issues make it an ineffective therapy. With the pneumococcal vaccine, a 2009 Cochrane review showed the overall incidence of AOM has reduced by 6% to 7%. Because as many as two-thirds of children who have influenza have AOM, studies have demonstrated the influenza vaccine has a 30% to 55% efficacy of preventing AOM during respiratory season. Breast-feeding for at least 4 to 6 months reduces episodes of AOM and recurrent AOM. In addition to other benefits, elimination of passive tobacco smoke exposure has been linked to a reduction of AOM in infancy.

A 42-year-old woman from Pakistan is being treated with infliximab for rheumatoid arthritis. After 6 months of therapy, she develops persistent fever, weight loss, and night sweats, and tuberculosis is suspected. Which of the following is the most likely location of the tuberculosis? A Middle and lower lung zones B Pleural space C Apical segment of the upper lung lobes D Cervical or supraclavicular lymph nodes

The correct answer is C. Explanation: Reactivation tuberculosis (in this case, likely triggered by infliximab) usually involves the apical aspects of the lungs. Primary pulmonary TB infection most often affects the middle and lower lobes. Lymphadenitis and pleural disease are the most common extrapulmonary TB infections, but they are less common than pulmonary TB.

A 39-year-old man develops a moderate free-flowing pleural effusion following a left lower lobe pneumonia. Thoracentesis reveals straw-colored fluid with gram-positive diplococci on Gram stain, pH 6.9, glucose 32 mg/dL, and LDH 1890. Which of the following is the best next step? A Send the fluid for culture. B Continue treatment with antibiotics for pneumococcal infection. C Tube thoracostomy to drain the effusion. D Schedule a follow-up chest x-ray in 2 weeks to document resolution of the effusion.

The correct answer is C. Explanation: The positive Gram stain, low pH, low glucose, and markedly elevated LDH all suggest that this parapneumonic effusion is "complicated," that is, it is unlikely to resolve with antibiotic therapy and is likely to produce loculated pockets of pus, which will require drainage with tube thoracostomy.

A 67-year-old long-time smoker with chronic obstructive pulmonary disease presents with 3 days of headaches and plethoric swelling of his face and right arm. Which of the following is the most likely diagnosis? A Angioedema B Hypothyroidism C Superior vena cava syndrome D Trichinosis

The correct answer is C. Explanation: The patient has features of SVC syndrome, caused by compression of the SVC, almost always by a thoracic malignancy. Urgent diagnosis and treatment are mandatory because of impaired cerebral venous drainage and resultant increased intracranial pressure or possibly fatal intracranial venous thrombosis. Angioedema, hypothyroidism, and trichinosis all may cause facial swelling, but not the plethora or swelling of the arm.

A 42-year-old woman who is known to be HIV positive is found to have a CD4 count of 125 cells/mm3 and is taking HAART. She has not experienced any AIDS-defining illness. She continues to use IV heroin and abuse alcohol on a daily basis. She does not regularly take her antiretroviral medication and is often lost to follow-up. Which of the following treatments is most appropriate at this time? A Initiate fluconazole for candidiasis prophylaxis. B Initiate antiviral treatment for herpes zoster prophylaxis. C Initiate TMP-SMX for P jiroveci pneumonia prophylaxis. D Initiate clarithromycin for M avium-intracellulare complex prophylaxis.

The correct answer is C. Explanation: With this level of cell count, the patient should continue antiretroviral therapy and start P jiroveci pneumonia prophylaxis. The level is not yet low enough to recommend M avium-intracellulare complex prophylaxis.

1. An 18-month-old girl is seen in the clinic for cough and fever. Her weight is in the third percentile. The mother reports she is concerned her daughter is around "toxic mold" because she has had five to six prior episodes of bronchitis since they moved to a new apartment at 6 months of age. She states that albuterol and an antibiotic are always given for treatment, and symptoms resolve in 2 weeks. A chest radiograph is obtained. What finding on the radiograph would prompt you to perform a sweat chloride test? a. An enlarged cardiac silhouette b. Absent thymus c. Bronchiectasis d. Dextrocardia e. Hilar lymphadenopathy

The correct answer is C. You answered C. Explanation: C. Bronchiectasis occurs as a sequela to impaired mucus clearance combined with inflammation and injury to the bronchial walls. CF is the most common noninfectious and chronic cause of this finding; sweat chloride testing is a standard for diagnosis. The other findings are not characteristics of CF.

1. A 7-year-old girl is admitted to the hospital in respiratory distress due to pneumonia. This is her third admission in the past 6 months. At this time you are suspecting cystic fibrosis (CF) and order a sputum culture. Which organism would be most consistent with a diagnosis of cystic fibrosis? a. Streptococcus pneumoniae b. Mycobacterium tuberculosis c. Pseudomonas aeruginosa d. Bacillus cereus e. Haemophilus influenza

The correct answer is C. You answered C. Explanation: C. The presence of P aeruginosa on a sputum sample strongly suggests the diagnosis of CF. Patients with CF have a high prevalence of colonization with P aeruginosa, Staphylococcus aureus, and Burkholderia cepacia. The innate defenses of the airway epithelium cells of CF patients may be compromised, making them unable to fight these organisms.

A 52-year-old man presents with dyspnea, and chest x-ray shows a hilar mass with ipsilateral pleural effusion. Which of the following is the best next step? A CT scan of the chest, head, and abdomen for cancer staging. B Pulmonary function testing to evaluate pulmonary reserve to evaluate for pulmonectomy. C Obtain a specific tissue diagnosis by biopsy of the hilar mass. D Initiate palliative radiation because the patient is not a candidate for curative resection.

The correct answer is C.however. farly thinks pulm would do a thorocentisis first Explanation: Tissue diagnosis is essential for proper treatment of any malignancy and should always be the first step. Once a specific tissue diagnosis is obtained, the cancer is staged for prognosis and to guide therapy, whether that is surgical resection, chemotherapy, or radiotherapy. Questions for this patient include the tissue type, location of spread, and whether the pleural effusion is caused by malignancy.

A 22-year-old woman tests positive for gonorrhea from routine screening during a well-woman examination. She was asymptomatic at the time of the testing. She has no known drug allergies. Which of the following treatments would be recommended for her? A Penicillin G 1.2 million units IM × 1 B Ceftriaxone 250 mg IM × 1 C Ciprofloxacin 250 mg PO × 1 dose D Ceftriaxone 250 mg IM × 1 and azithromycin 1 g PO × 1

The correct answer is D. Explanation: Patients who test positive for gonorrhea should be treated for both gonorrhea and Chlamydia. Ceftriaxone 250 mg IM is the appropriate treatment for gonorrhea and azithromycin is the appropriate treatment for Chlamydia. Her sexual partner(s) should also be offered treatment.

A 5-year-old girl developed high fever, ear pain, and vomiting a week ago. She was diagnosed with OM and started on amoxicillin-clavulanate. On the third day of this medication she continued with findings of OM, fever, and pain. She received ceftriaxone intramuscularly and switched to oral cefuroxime. Now, 48 hours later, she has fever, pain, and no improvement in her OM; otherwise she is doing well. Which of the following is the most logical next step in her management? a. Addition of intranasal topical steroids to the oral cefuroxime b. Adenoidectomy c. High-dose oral amoxicillin d. Oral trimethoprim-sulfamethoxazole e. Tympanocentesis and culture of middle ear fluid

The correct answer is E. E. After failing several antibiotic regimens, tympanocentesis and culture of the middle ear fluid are indicated.

A 34-year-old man with a past history of asthma presents to an acute care clinic with an asthma exacerbation. Treatment with nebulized albuterol and ipratropium does not offer significant improvement, and he is then admitted to the hospital. He is afebrile, has a respiratory rate of 24 breaths/min, pulse rate is 96 beats/min, and oxygen saturation is 93% on room air. On examination, he has diffuse bilateral inspiratory and expiratory wheezes, mild intercostal retractions, and a clear productive cough. Which one of the following should be the next step in the management of this patient? A Chest physical therapy B Inhaled corticosteroids C Azithromycin orally D Theophylline orally E Oral corticosteroids

The correct answer is E. Explanation: Hospital management of acute exacerbations of asthma should include inhaled short-acting bronchodilators and systemic corticosteroids. The efficacy of oral versus intravenous corticosteroids has been shown to be equivalent. Antibiotics are not needed in the treatment of asthma exacerbations unless there are signs of infection. Inhaled ipratropium is recommended for treatment in the emergency department, but not in the hospital. Chest physical therapy and theophylline are not recommended for acute asthma exacerbations.

A 13-year-old adolescent boy has a nonproductive cough and mild shortness of breath on a daily basis. He is awakened by the cough at least five nights per month. Which one of the following would be the most appropriate treatment for this patient? A A long-acting β-agonist daily B A short-acting β-agonist daily C Oral prednisone daily D An oral leukotriene inhibitor as needed E Inhaled corticosteroids daily

The correct answer is E. Explanation: This patient has moderate persistent asthma. The most effective treatment is daily inhaled corticosteroids. A leukotriene inhibitor would be less effective and as a controller should be used daily. Oral prednisone daily is problematic due to the risk of adrenal insufficiency. Short- and long-acting β-agonists are not recommended as daily therapy because they are considered rescue medications rather than asthma controllers.

A 25-year-old previously healthy man presents to the emergency room after experiencing a generalized tonic-clonic seizure that lasted 30 seconds. He has been experiencing headaches over the past 6 months but no other associated symptoms. His mother states that she witnessed him to have two previous seizures. The patient has a history of being sexually promiscuous and using IV illicit drugs. The result of his last HIV test is unknown. On neurologic examination, he is noted to have increased tone on the right and decreased right arm swing when walking. The remainder of his neurologic examination is unremarkable. A computed tomography (CT) scan of the head with contrast reveals that he has a ring-enhancing lesion measuring 15-mm over the left motor strip region and a 12-mm ring-enhancing lesion in the left basal ganglia. Which of the following would be an AIDS-defining condition in this patient? A Glioblastoma multiforme B Subarachnoid hemorrhage C Herpes zoster encephalitis D Listeriosis with brain abscess E Primary brain lymphoma

The correct answer is E. Explanation: Primary brain lymphoma is an AIDS-defining condition. Glioblastoma multiforme and subarachnoid hemorrhage may present with these symptoms, but are not AIDS-defining conditions. Listeriosis and herpes zoster encephalitis can be associated with HIV, but are not AIDS-defining conditions.

A 7-year-old boy is brought to a hospital in Charlotte, North Carolina with a fever of 104.0°F (40.0°C). A maculopapular rash is seen on his wrists and ankles but the palms and soles are spared. His laboratory results show leukopenia, hyponatremia, and elevated liver transaminases. His parents say that he was on a camping trip 1 week ago but they vigorously used insect repellants and filtered all of their water. His father came in contact with poison oak, but the boy denies any pruritus. Which of the following is the best treatment for this patient's rash? A. Penicillin B. Acyclovir C. Ceftriaxone D. Vancomycin E. Doxycycline

The correct answer is E. You answered E. Explanation: The patient has Rocky Mountain spotted fever and should be treated with doxycycline. The disease is commonly found in North Carolina and is carried by ticks that the boy could have picked up during the camping trip. RMSF has a characteristic rash that starts on the wrists and ankles and can eventually involve the palms and soles. Typically, the rash of RMSF spreads centripetally from the wrists and ankles to involve the trunk and extremities.

A 22-year-old African-American woman presents with fatigue, arthralgias, and a nagging dry cough for the past 6 weeks, but no shortness of breath. On physical examination, her lungs are clear to auscultation, and she has bilateral pretibial tender erythematous raised nodules. Which of the following is your best next step? a.. Chest radiograph b.High-resolution CT c.Empiric treatment for postnasal drip d.Antinuclear antibody e.Initiation of antituberculosis therapy

a. chest radiograph The patient has clinical features suggestive of sarcoidosis given the new cough, arthralgias, and description of erythema nodosum. The initial, most cost-effective study is a chest radiograph. Hilar lymphadenopathy with or without interstitial infiltrates would solidify a diagnosis of sarcoidosis. A high-resolution CT may be ordered if the patient has interstitial lung disease, but it is not the first study of choice. Postnasal drip does not explain the patient's other symptoms. An antinuclear antibody would not necessarily identify the cause of the cough or provide a diagnosis.

If a child with AOM has received amoxicillin in the previous 30 days, has a history of recurrent AOM unresponsive to amoxicillin, or has concurrent purulent conjunctivitis, what abx should be given

an antibiotic with β-lactamase coverage is warranted

How is reversibility defined

an increase in FEV1 of greater than 12% or 200mL

A 2½-year-old boy comes to your clinic for the first time with complaints of fever and increasing "wet" cough for 8 days. His mother reports that he has been diagnosed with asthma and has an albuterol inhaler to use for wheezing or cough. Since 6 months of age, he has had several similar episodes of "wet" cough and fever, which were diagnosed as bronchitis or pneumonia, and he would improve when treated with antibiotics and albuterol. However, over the past year, these episodes have become more frequent and the cough occurs almost daily now. Sometimes the mother sees him expectorate the sputum, which is thick and purulent. He has daily nasal congestion for which she uses saline and bulb suction in his nares. She is able to obtain some thick yellow discharge but the symptoms mainly improve when he is treated with antibiotics. He is not in daycare and has no tobacco exposure. She is concerned that his frequent illnesses are causing him to be "small for his age." The mother notes his stools are malodorous, and since starting him on potty-training she has observed that his stools float and sometimes appear to have drops of oil on them. Your examination reveals a moderately ill-appearing child whose height and weight are at the third percentile for age. His temperature is 101°F (38.3°C) and respiratory rate is 32 breaths/min. He is breathing with his mouth open. Over the upper lung fields, he has crackles and rhonchi and also a few expiratory wheezes over all lung fields. He has no heart murmur; S1 and S2 are normal. His fingers show clubbing. You obtain a chest radiograph that shows linear opacities in a parallel tram-track configuration in the upper lobes with some ring-shaped opacities; the radiologist interprets the findings as bronchiectasis. What is the most likely diagnosis

cystic fibrosis

1.Which of the following is most accurate? a.Cough caused by captopril may resolve with switching to enalapril. b.Initial treatment of a chronic cough should include codeine or a similar opiate derivative to suppress the cough. c.Cough caused by reflux can be effectively ruled out by a negative history of heartburn or dyspepsia. d.More than one condition is often responsible for causing chronic cough in a given patient.

d.More than one condition is often responsible for causing chronic cough in a given patient. Often more than one condition is responsible for causing chronic cough in a given patient. Cough from ACE inhibitors is class dependent, and change to another class of antihypertensives is more appropriate. The etiology of chronic cough should be determined prior to suppression of the cough because treatment of the underlying condition is the most effective approach. A patient with GERD may present with the sole manifestation of cough, or it may present "silently."

How often should pts who are immunocompromised or without a functioning spleen be revacinnated with the penumococcal vaccine

every 5 years

When should supplemental O2 be given

if the pts O2 sat is around or less than 92% and based on how they present

What other vaccine besides the penumococcal vaccine is recommended for all individuals aged 6 months and oldre

influenza vaccination

What is the "gold standard" to dx asthma

methacholine challenge

Signs and symptoms for patients at presentation with symptoms who are particularly at risk for poor control of the disease and outcomes including death include what?

oA chest examination with minimal sounds on auscultation oDistress and difficulty speaking on examination oTachycardia oExtremely low peak flow (<50% of predicted) oElevated carbon dioxide on arterial blood gas with hypoxia. Typically, arterial carbon dioxide is low during an asthmatic attack due to increased respiratory rate. Normalization of carbon dioxide may be a sign of a severe exacerbation indicating CO2 retention and impending respiratory failure.

Signs and symptoms for patients at regular ambulatory visits who are particularly at risk for poor control of the disease and outcomes including death include what?

oPrevious admission for intensive care or intubations oThree or more emergency department visits for asthma in the last year oTwo or more canisters of short-acting β-agonists in a month oFailure to use controllers (inhaled corticosteroids) despite symptoms oCurrent or recent cessation of oral corticosteroids oLarge fluctuations in peak flow oLow socioeconomic status in an urban environment oMental disorders or substance abuse

After a pt is treated with UFH, LMWH, etc (initial anticoaculation therapy), what anticoagulation can they then be started on

warfarin

A 52-year-old man presents because of coughing and SOB. He has had multiple office visits in the past few years for similar reasons. He has a chronic "smoker's cough," but reports that in the past 2 days his cough has increased, his sputum has changed from white to green in color, and he has had to increase the frequency with which he uses his albuterol inhaler. He denies having a fever, CP, peripheral edema, or other symptoms. His medical history is significant for hypertension, peripheral vascular disease, and two hospitalizations for pneumonia in the past 5 years. He has a 60-pack-year history of smoking and continues to smoke two packs of cigarettes a day. On examination, he is in moderate respiratory distress. His temperature is 98.4°F, his blood pressure is 152/95 mm Hg, his pulse is 98 beats/min, his respiratory rate is 24 breaths/min, and he has an oxygen saturation of 94% on room air. His lung examination is significant for diffuse expiratory wheezing and a prolonged expiratory phase of respiration. There are no signs of cyanosis. The remainder of his examination is normal. A chest x-ray done in your office shows an increased anteroposterior (AP) diameter and a flattened diaphragm, but otherwise he has clear lung fields What acute treatments are most appropriate at this time

•Albuterol nebulizer (SABA) •Corticosteroid (route of administration varies based on severity) •Antibiotics •Ipratropium bromide (anticholinergic) •O2 if needed (<90%)

What are some examples of AIDS defining conditions

•Candidiasis of bronchi, trachea, or lungs •Coccidioidomycosis (disseminated or extrapulmonary) •Cytomegalovirus disease •Disseminated or extrapulmonary histoplasmosis •Burkitt lymphoma •M avium complex (disseminated or extrapulmonary) •Pneumonia, recurrent •Toxoplasmosis of brain •Esophageal candidiasis •Extrapulmonary Cryptococcus •HIV-related encephalopathy •Intestinal isosporiasis (>1-mo duration) •Immunoblastic lymphoma •Mycobacterium tuberculosis (any site) •Progressive multifocal leukoencephalopathy •Wasting syndrome caused by HIV •Invasive cervical cancer •Intestinal cryptosporidiosis (>1-mo duration) •Herpes simplex: chronic ulcer, bronchitis, pneumonitis, or esophagitis •Kaposi sarcoma •*Primary brain lymphoma* •*P jiroveci pneumonia* Recurrent Salmonella septicemia

What is a transudative effusion? What are the LDH and protein levels like? What are some common causes

•Effusion caused by alteration of oncotic forces, usually with low protein and low lactate dehydrogenase (LDH) levels. oCauses: CHF is the most common cause, PE, cirrhosis, nephrotic syndrome, atelectasis bc it creates a vacuum effect

In a pleural effusion, what characteristics suggest the need for chest tube drainage

•Empyema (frank pus in the pleural space) •Positive Gram stain or culture of fluid •Presence of loculations •pH less than 7.20 (pleural fluid pH is normally alkaline with pH of 7.6) •Glucose less than 60 mg/dL •LDH more than 1000 U/L

Emergency treatment should be considered in asthma pts in what situations

•Peak flow less than 40% of predicted normal (based on age, gender, and height) •Failure to respond to a β2-agonist •Severe wheezing or coughing •Extreme anxiety due to breathlessness •Gasping for air, sweaty, or cyanotic •Rapid clinical deterioration over a few hours with hypoxia •Severe retractions and nasal flaring •Posture with shoulders hunched forward

What are some symptoms of CF

•recurrent wet cough, thick sputum, daily nasal congestion, maloderous oily stool, failure to thrive, bronchiectasis on CXR -wheezing and cough that mimic asthma and bronchiolitis , meconium ileus --> infant does not pass stoll


Conjuntos de estudio relacionados

accounting Which of the following is not an external users of financial accounting information?

View Set

KN 136 strength assessment and explosive movements

View Set

Renaissance and Mannerism in Cinquecento Italy

View Set

Project Manager (Quality and Safety)

View Set